FirstRanker Logo

FirstRanker.com - FirstRanker's Choice is a hub of Question Papers & Study Materials for B-Tech, B.E, M-Tech, MCA, M.Sc, MBBS, BDS, MBA, B.Sc, Degree, B.Sc Nursing, B-Pharmacy, D-Pharmacy, MD, Medical, Dental, Engineering students. All services of FirstRanker.com are FREE

📱

Get the MBBS Question Bank Android App

Access previous years' papers, solved question papers, notes, and more on the go!

Install From Play Store

NEET PG 2023 Question Paper With Solutions

Master NEET PG with 15 Years of Solved Question Papers (2012-2025) --NEET PG 2023 Question Paper With Solutions

This post was last modified on 25 May 2025


NEET PG 2023 PYQS

Here is a valuable resource to elevate your NEET PG preparation - the NEET PG 2023
Previous Year Questions PDF. These recal -based questions have been careful y vetted by

--- Content provided by FirstRanker.com ---

India's leading medical faculty to ensure accuracy and relevance.
You can use this PDF to strengthen your exam preparations by identifying and working on the
high-yield topics that have been asked repeatedly over the past years.
Also, if you're looking to enhance your understanding further of these topics, download the
PrepLadder app and get access to QBank 5.0 for detailed explanations for al correct and

--- Content provided by FirstRanker.com ---

incorrect options. You wil also get a ton of other excel ent preparation resources to help you
stay ahead in your journey to success! Find the download link at the end.
Ques No: 1

Subject:
Microbiology

--- Content provided by FirstRanker.com ---


Topic:
Virology

Sub-Topic:

A patient hailing from Delhi presents with fever, arthralgia, and extensive petechial rash for 3

--- Content provided by FirstRanker.com ---

days. Lab investigations revealed a hemoglobin of 9 g/ dL, a white blood cel count of 9000
cel s/mm3, a platelet count of 20000 cel s/mm3, and a prolonged bleeding time. The clotting
time was normal. What is the most likely diagnosis?

O1:

--- Content provided by FirstRanker.com ---

Dengue

O2:

Malaria

--- Content provided by FirstRanker.com ---

O3:

Scrub typhus

O4:

Typhoid

--- Content provided by FirstRanker.com ---


Ans: 1

Ques No: 2

Subject:
Mi crobiology

--- Content provided by FirstRanker.com ---


Topic:
Parasitology

Sub-Topic:

Which of the fol owing statements is true about Trichomonas vaginalis?

--- Content provided by FirstRanker.com ---


O1:

It cannot be cultured

O2:

--- Content provided by FirstRanker.com ---

Twitching motility is seen on wet saline mount


O3:

Cysts are seen on wet saline mount of vaginal secretions

--- Content provided by FirstRanker.com ---

O4:

It is not a sexual y transmitted infection

Ans: 2

Ques No: 3

--- Content provided by FirstRanker.com ---


Subject:
Microbiology

Topic:
Parasitology

--- Content provided by FirstRanker.com ---

Sub-Topic:

A lady from West Rajasthan presented with an ulcer surrounded by erythema on the right leg.
Microscopy of the biopsy from the edge of the ulcer showed organisms with dark staining
nuclei and kinetoplast. What is the most likely causative agent?

--- Content provided by FirstRanker.com ---

O1:

Leishmania tropica

O2:

Babesia

--- Content provided by FirstRanker.com ---


O3:

Trypanosoma

O4:

--- Content provided by FirstRanker.com ---

Histoplasma

Ans: 1

Ques No: 4

--- Content provided by FirstRanker.com ---

Subject:
Microbiology

Topic:
Virology

Sub-Topic:

--- Content provided by FirstRanker.com ---

A child presents with a fever and a rash. Urine examination showed cel s with owl's eye
appearance. What is the most likely diagnosis?

O1:

Cytomegalovirus

--- Content provided by FirstRanker.com ---


O2:

Epstein-Barr virus

O3:

--- Content provided by FirstRanker.com ---

Herpes simplex virus

O4:

Toxoplasma gondi

--- Content provided by FirstRanker.com ---

Ans: 1

Ques No: 5

Subject:
Microbiology

--- Content provided by FirstRanker.com ---

Topic:
Parasitology


Sub-Topic:

Infection with Clonorchis sinensis is associated with an increased risk of__?

--- Content provided by FirstRanker.com ---

O1:

Cervical cancer

O2:

Cholangiocarcinoma

--- Content provided by FirstRanker.com ---


O3:

Gastric carcinoma

O4:

--- Content provided by FirstRanker.com ---

Bladder carcinoma

Ans: 2

Ques No: 6

--- Content provided by FirstRanker.com ---

Subject:
Microbiology

Topic:
Mycology

Sub-Topic:

--- Content provided by FirstRanker.com ---

A patient presented with complaints of persistent cough and weight loss. He had a history of
contact with multiple sexual partners. He was diagnosed as HIV positive 6 months back and is
poorly adherent to his medications. The fungal culture of sputum was negative. On Gomori
methamine silver staining, the cel s showed darkly stained crescent-shaped cysts. What is the
most likely causative organism?

--- Content provided by FirstRanker.com ---


O1:

Paracoccidiodes brasiliensis

O2:

--- Content provided by FirstRanker.com ---

Histoplasma capsulatum

O3:

Coccidiodes immitis

--- Content provided by FirstRanker.com ---

O4:

Pneumocyst is jerovecii

Ans: 4

Ques No: 7

--- Content provided by FirstRanker.com ---


Subject:
Microbiology

Topic:
Systemic Bacteriology

--- Content provided by FirstRanker.com ---

Sub-Topic:

A sewage worker presented with abdominal pain, jaundice, conjunctival injection, and blood in
the urine for the past 5 days. Which of the fol owing is the investigation of choice?

O1:

--- Content provided by FirstRanker.com ---

Widal test

O2:

Microscopic agglutination test

--- Content provided by FirstRanker.com ---

O3:



Weil-Felix reaction

O4:

--- Content provided by FirstRanker.com ---

Paul-Bunnel test

Ans: 2

Ques No: 8

--- Content provided by FirstRanker.com ---

Subject:
Microbiology

Topic:
Virology

Sub-Topic:

--- Content provided by FirstRanker.com ---

Which of the fol owing laboratory test is used to diagnose rotavirus diarrhea?

O1:

Antigen detection in the stool sample

--- Content provided by FirstRanker.com ---

O2:

Antigen detection in blood

O3:

Antibody detection in serum

--- Content provided by FirstRanker.com ---


O4:

Light microscopy of stool specimen

Ans: 1

--- Content provided by FirstRanker.com ---

Ques No: 9

Subject:
Microbiology

Topic:
Parasitology

--- Content provided by FirstRanker.com ---


Sub-Topic:

A boy presented with a fever and chil s. Rapid test was positive for specific antigen HRP-2.
Which of the fol owing species of Plasmodium is the most likely causative agent?

--- Content provided by FirstRanker.com ---

O1:

Plasmodium falciparum

O2:

Plasmodiu m malariae

--- Content provided by FirstRanker.com ---


O3:

Plasmodium vivax

O4:

--- Content provided by FirstRanker.com ---

Plasmodium ovale

Ans: 1

Ques No: 10

--- Content provided by FirstRanker.com ---

Subject:
Microbiology

Topic:
Systemic Bacteriology

Sub-Topic:

--- Content provided by FirstRanker.com ---

A child was brought with complaints of high fever, multiple seizures, headache, and neck


rigidity. CSF analysis showed low glucose, high protein, and many polymorphs. The CSF
culture showed pleomorphic gram-negative bacil i. What is true about the causative organism?

--- Content provided by FirstRanker.com ---

O1:

It is bacitracin sensitive.

O2:

Produces non-lactose fermenting colonies on MacConkey agar

--- Content provided by FirstRanker.com ---


O3:

Exhibits satel itism around Staphylococcus aureus colonies

O4:

--- Content provided by FirstRanker.com ---

It exhibits alpha hemolysis on sheep blood agar

Ans: 3

Ques No: 11

--- Content provided by FirstRanker.com ---

Subject:
Microbiology

Topic:
Systemic Bacteriology

Sub-Topic:

--- Content provided by FirstRanker.com ---

A man presents with a history of dysuria and urethral discharge after having unprotected
sexual intercourse. Gram-staining of the purulent discharge is seen in the fol owing image.
What is the treatment of choice for this infection?

O1:

--- Content provided by FirstRanker.com ---

Erythromyci n

O2:

Azithromycin

--- Content provided by FirstRanker.com ---

O3:

Ceftriaxone

O4:

Penicil in G

--- Content provided by FirstRanker.com ---


Ans: 3



Ques No: 12

--- Content provided by FirstRanker.com ---

Subject:
Radiology

Topic:
Head & Neck Imaging

Sub-Topic:

--- Content provided by FirstRanker.com ---

A 2-month-old infant is brought to the OPD with a parietal swel ing present since birth. The
X-ray image is given below. What could be the probable diagnosis?

O1:

Subgaleal hematoma

--- Content provided by FirstRanker.com ---


O2:

Cephalhematoma

O3:

--- Content provided by FirstRanker.com ---

Caput succedaneum

O4:

Enecephalo cele

--- Content provided by FirstRanker.com ---

Ans: 2

Ques No: 13

Subject:
Radiology

--- Content provided by FirstRanker.com ---

Topic:
Genitourinary Radiology

Sub-Topic:

A patient presented with abdominal pain and sterile pyuria, and the x-ray showed the fol owing
features. What is the likely diagnosis?

--- Content provided by FirstRanker.com ---




O1:

Putty kidney

--- Content provided by FirstRanker.com ---

O2:

Nephrocalcinosis

O3:

Staghorn calculus

--- Content provided by FirstRanker.com ---


O4:

Psoas calcification

Ans: 1

--- Content provided by FirstRanker.com ---

Ques No: 1 4

Subject:
Anatomy

Topic:
Pelvis and Perineum

--- Content provided by FirstRanker.com ---


Sub-Topic:

A defect in which of the fol owing forms the structure marked below?



--- Content provided by FirstRanker.com ---

O1:

Internal oblique

O2:

External oblique

--- Content provided by FirstRanker.com ---


O3:

Parietal peritoneum

O4:

--- Content provided by FirstRanker.com ---

Fascia transversalis

Ans: 4

Ques No: 1 5

--- Content provided by FirstRanker.com ---

Subject:
Anatomy

Topic:
Head and Neck

Sub-Topic:

--- Content provided by FirstRanker.com ---

A student had his jaw locked while yawning. Which of the fol owing muscles is attached to the
articular disc of the temporomandibular joint?

O1:

Lateral pterygoid

--- Content provided by FirstRanker.com ---


O2:

Temporalis

O3:

--- Content provided by FirstRanker.com ---

Medial pterygoid



O4:

Masseter

--- Content provided by FirstRanker.com ---


Ans: 1

Ques No: 16

Subject:
Anatomy

--- Content provided by FirstRanker.com ---


Topic:
Histology

Sub-Topic:

The given histology image is of ?

--- Content provided by FirstRanker.com ---


O1:

Leydig cel s of testis

O2:

--- Content provided by FirstRanker.com ---

Pancreatic islet cel s

O3:

Hassal 's corpuscles

--- Content provided by FirstRanker.com ---

O4:

Glomerulus

Ans: 4

Ques No: 1 7

--- Content provided by FirstRanker.com ---


Subject:
Anatomy

Topic:
Thorax

--- Content provided by FirstRanker.com ---

Sub-Topic:

A defect in which of the fol owing aortic arches causes the defect shown in the image?



O1:

--- Content provided by FirstRanker.com ---

4th right

O2:

4th left

--- Content provided by FirstRanker.com ---

O3:

6th right

O4:

6th left

--- Content provided by FirstRanker.com ---


Ans: 4

Ques No: 18

Subject:
Anatomy

--- Content provided by FirstRanker.com ---


Topic:
General Embryology

Sub-Topic:

Which of the fol owing structures develops from the structure marked 4?

--- Content provided by FirstRanker.com ---


O1:

Medial umbilical ligament

O2:

--- Content provided by FirstRanker.com ---

Median umbilical ligament



O3:

Meckel's diverticulum

--- Content provided by FirstRanker.com ---


O4:

Ligamentum teres

Ans: 2

--- Content provided by FirstRanker.com ---

Ques No: 19

Subject:
Anatomy

Topic:
Abdomen

--- Content provided by FirstRanker.com ---


Sub-Topic:

A patient comes with abdominal pain, jaundice, and portal hypertension. Anastomosis
between which of the fol owing veins is seen?

--- Content provided by FirstRanker.com ---

O1:

Left colic vein and middle colic veins

O2:

Esophageal veins and left gastric veins

--- Content provided by FirstRanker.com ---


O3:

Superior rectal and phrenic veins

O4:

--- Content provided by FirstRanker.com ---

Sigmoid and superior rectal veins

Ans: 2

Ques No: 20

--- Content provided by FirstRanker.com ---

Subject:
Biochemistry

Topic:
Vitamins

Sub-Topic:

--- Content provided by FirstRanker.com ---

Which of the fol owing clotting factors is needed for gamma-glutamyl carboxylase?


O1:

Factors II, VIII, IX, X

--- Content provided by FirstRanker.com ---

O2:

Factors II, VII, IX, and X

O3:

Factors II, VII, IX, XI

--- Content provided by FirstRanker.com ---


O4:

Factors II, VIII, X, XI

Ans: 2

--- Content provided by FirstRanker.com ---

Ques No: 21

Subject:
Biochemistry

Topic:
Vitamins

--- Content provided by FirstRanker.com ---


Sub-Topic:

A patient presented with diarrhea, dermatitis, and dementia. Which of the fol owing vitamin
deficiencies would you suspect in this patient?

--- Content provided by FirstRanker.com ---

O1:

Vitamin B3 deficiency

O2:

Vitamin B1 deficiency

--- Content provided by FirstRanker.com ---


O3:

Vitamin B6 deficiency

O4:

--- Content provided by FirstRanker.com ---

Vitamin B12 deficiency

Ans: 1

Ques No: 22

--- Content provided by FirstRanker.com ---

Subject:
Biochemistry

Topic:
Carbohydrate Metabolism

Sub-Topic:

--- Content provided by FirstRanker.com ---

A young bo y presents to the OPD with hypoglycemia and muscle cramps, on exertion or while
playing. Then he becomes normal after resting for a while. These episodes are recurrent after
a period of activity. He has decreased serum lactate and glucose levels. Which of the fol owing
diseases is he most likely to be suf ering from?

--- Content provided by FirstRanker.com ---

O1:

McArdle disease

O2:

Hers disease

--- Content provided by FirstRanker.com ---


O3:

Cori's disease

O4:

--- Content provided by FirstRanker.com ---

Andersen disease


Ans: 1

Ques No: 23

--- Content provided by FirstRanker.com ---

Subject:
Biochemistry

Topic:
Vitamins

Sub-Topic:

--- Content provided by FirstRanker.com ---

A family consumes only polished rice. Which of the fol owing combinations of vitamin
deficiency and enzymatic defect wil be present in this family?

O1:

Riboflavin-glutathione reductase

--- Content provided by FirstRanker.com ---


O2:

Thiamine-transketolase

O3:

--- Content provided by FirstRanker.com ---

Thiamine-transaminase

O4:

Riboflavin-transketolase

--- Content provided by FirstRanker.com ---

Ans: 2

Ques No: 24

Subject:
Biochemistry

--- Content provided by FirstRanker.com ---

Topic:
Vitamins

Sub-Topic:

A chronic alcoholic patient presented to the casualty with confusion, ataxia, and painful eye
movements. What could be the vitamin deficiency seen in this patient?

--- Content provided by FirstRanker.com ---


O1:

Vitamin B deficiency

O2:

--- Content provided by FirstRanker.com ---

Vitamin A deficiency

O3:

Vitamin D d eficiency

--- Content provided by FirstRanker.com ---

O4:

Vitamin C deficiency

Ans: 1

Ques No: 25

--- Content provided by FirstRanker.com ---


Subject:
Biochemistry

Topic:
Carbohydrate Metabolism

--- Content provided by FirstRanker.com ---

Sub-Topic:

A 4-year-old child presents with easy fatigability. The mother also complains that the child has
increased hunger between meals, which is relieved after food. Liver examination revealed no
glycogen. The enzyme most likely deficient is?

--- Content provided by FirstRanker.com ---

O1:



Glycogen phosphorylase

O2:

--- Content provided by FirstRanker.com ---

Glycogen synthase

O3:

Debranching enzyme

--- Content provided by FirstRanker.com ---

O4:

Glucose 6 phosphatase

Ans: 2

Ques No: 26

--- Content provided by FirstRanker.com ---


Subject:
Biochemistry

Topic:
Vitamins

--- Content provided by FirstRanker.com ---

Sub-Topic:

A child was bought with pedal edema and cheilosis. Cardiomegaly was present. What is the
vitamin deficiency associated with this clinical presentation?

O1:

--- Content provided by FirstRanker.com ---

Riboflavin

O2:

Thiamine

--- Content provided by FirstRanker.com ---

O3:

Pyridoxine

O4:

Niacin

--- Content provided by FirstRanker.com ---


Ans: 2

Ques No: 27

Subject:
Biochemistry

--- Content provided by FirstRanker.com ---


Topic:
Vitamins

Sub-Topic:

A chronic al coholic patient is diagnosed with gouty arthritis. What biochemical changes wil be

--- Content provided by FirstRanker.com ---

seen in this patient?

O1:

Decreased NADH/NAD+ ratio

--- Content provided by FirstRanker.com ---

O2:

Increased urea and urate levels

O3:

Increased urate and lactate levels

--- Content provided by FirstRanker.com ---


O4:

Increased level of alkaline phosphatase

Ans: 3

--- Content provided by FirstRanker.com ---

Ques No: 28


Subject:
Biochemistry

Topic:
Enzymes

--- Content provided by FirstRanker.com ---


Sub-Topic:

A child presents with erythematous scaly patches in the perioral region, mucosal ulcers, and
impaired epithelial wound healing. The most likely mineral deficiency associated with this
condition is?

--- Content provided by FirstRanker.com ---


O1:

Iron deficiency

O2:

--- Content provided by FirstRanker.com ---

Zinc deficiency

O3:

Calcium deficiency

--- Content provided by FirstRanker.com ---

O4:

Copper deficiency

Ans: 2

Ques No: 29

--- Content provided by FirstRanker.com ---


Subject:
Biochemistry

Topic:
Vitamins

--- Content provided by FirstRanker.com ---

Sub-Topic:

A patient presenting with bleeding gums and easy bruisability was diagnosed with scurvy. This
is due to

O1:

--- Content provided by FirstRanker.com ---

Low calcium

O2:

Defective col agen formation

--- Content provided by FirstRanker.com ---

O3:

Inhibition of clotting factors

O4:

Increased keratinization of epithelium

--- Content provided by FirstRanker.com ---


Ans: 2

Ques No: 30

Subject:
Biochemistry

--- Content provided by FirstRanker.com ---


Topic:
Introduction

Sub-Topic:

Which of the fol owing is seen in low insulin: glucagon ratio?

--- Content provided by FirstRanker.com ---


O1:

Activation of lipoprotein lipase

O2:

--- Content provided by FirstRanker.com ---

Activation of glycogen synthase

O3:



Activation of phosphofructokinase 1

--- Content provided by FirstRanker.com ---


O4:

Activation of hormone-sensitive lipase

Ans: 4

--- Content provided by FirstRanker.com ---

Ques No: 31

Subject:
Biochemistry

Topic:
Amino Acids and Proteins Chemistry

--- Content provided by FirstRanker.com ---


Sub-Topic:

An infant is brought by his parents with complaints that his urine turns black on standing.
Which of the fol owing metabolic disorders is likely?

--- Content provided by FirstRanker.com ---

O1:

Phenylketonuria

O2:

Alkaptonuria

--- Content provided by FirstRanker.com ---


O3:

Homocystinuria

O4:

--- Content provided by FirstRanker.com ---

Maple syrup urine disease

Ans: 2

Ques No: 32

--- Content provided by FirstRanker.com ---

Subject:
Biochemistry

Topic:
Vitamins

Sub-Topic:

--- Content provided by FirstRanker.com ---

A child presents with thinning of hair and scaling of the scalp. On taking further history, it is
found that she consumes raw eggs in her diet regularly. The given manifestations are due to
the deficiency of which of the fol owing vitamins?

O1:

--- Content provided by FirstRanker.com ---

Niacin

O2:

Pyridoxine

--- Content provided by FirstRanker.com ---

O3:

Biotin

O4:

Thiamine

--- Content provided by FirstRanker.com ---


Ans: 3

Ques No: 33

Subject:
Biochemistry

--- Content provided by FirstRanker.com ---


Topic:
Lipid Metabolism

Sub-Topic:



--- Content provided by FirstRanker.com ---

A child presents with mental retardation, bone pain, and inability to walk. On funduscopic
examination, a cherry red spot is seen. There is no organomegaly. What is the most likely
diagnosis?

O1:

--- Content provided by FirstRanker.com ---

Tay-Sachs disease

O2:

Niemann-Pick disease

--- Content provided by FirstRanker.com ---

O3:

Hurler syndrome

O4:

Gaucher disease

--- Content provided by FirstRanker.com ---


Ans: 1

Ques No: 34

Subject:
Ophthalmology

--- Content provided by FirstRanker.com ---


Topic:
Neuro Ophthalmology

Sub-Topic:

A female presents with loss of vision in the right halves of both eyes. Where is the lesion

--- Content provided by FirstRanker.com ---

located in the optic pathway?

O1:

Left optic tract

--- Content provided by FirstRanker.com ---

O2:

Optic radiation

O3:

Optic chiasma

--- Content provided by FirstRanker.com ---


O4:

Right optic tract

Ans: 1

--- Content provided by FirstRanker.com ---

Ques No: 3 5

Subject:
Ophthalmology

Topic:
Diseases of Lens

--- Content provided by FirstRanker.com ---


Sub-Topic:

A 65-year-old diabetic woman presents with painless loss of vision. The ocular findings are
shown in the image below. What is the likely diagnosis and management for this patient?


--- Content provided by FirstRanker.com ---



O1:

Zonular cataract and lensectomy

--- Content provided by FirstRanker.com ---

O2:

Rosette cataract and sugar control

O3:

Sunflower cataract and lesion extraction

--- Content provided by FirstRanker.com ---


O4:

Immature senile cataract and phacoemulsification with IOL

Ans: 4

--- Content provided by FirstRanker.com ---

Ques No: 36

Subject:
Ophthalmology

Topic:
Sclera

--- Content provided by FirstRanker.com ---


Sub-Topic:

A 5-year-old boy presents with a smal nodule on the sclera as shown in the image below.
What is the likely diagnosis?

--- Content provided by FirstRanker.com ---

O1:



Pterygium

O2:

--- Content provided by FirstRanker.com ---

Pinguecula

O3:

Dermolipoma

--- Content provided by FirstRanker.com ---

O4:

Dermoid

Ans: 4

Ques No: 37

--- Content provided by FirstRanker.com ---


Subject:
Ophthalmology

Topic:
Cornea

--- Content provided by FirstRanker.com ---

Sub-Topic:

The eye examination of a 20-year-old male shows a golden ring on the iris. What is the next
investigation to diagnose the condition?

O1:

--- Content provided by FirstRanker.com ---

Alpha-1-antitrypsin

O2:

Alpha-fetoprotein

--- Content provided by FirstRanker.com ---

O3:

Serum iron

O4:

Serum ceruloplasmin

--- Content provided by FirstRanker.com ---


Ans: 4

Ques No: 38

Subject:
Ophthalmology

--- Content provided by FirstRanker.com ---


Topic:
Cornea

Sub-Topic:

What is the dye and filter used to diagnose corneal ulcers?

--- Content provided by FirstRanker.com ---


O1:

Lissamine dye green filter

O2:

--- Content provided by FirstRanker.com ---

Lissamine dye cobalt blue filter

O3:

Fluorescein dye - visualized under green filter

--- Content provided by FirstRanker.com ---

O4:

Fluorescein dye - visualized under cobalt blue filter

Ans: 4

Ques No: 39

--- Content provided by FirstRanker.com ---




Subject:
Surgery

Topic:
Urology

--- Content provided by FirstRanker.com ---


Sub-Topic:

A 55-year-old male presented with verrucous carcinoma around the glans of the penis.
Examination reveals that the inguinal lymph nodes are not enlarged. What is the appropriate
management for this patient?

--- Content provided by FirstRanker.com ---


O1:

Total penectomy

O2:

--- Content provided by FirstRanker.com ---

CO2 laser excision

O3:

Topical 5-fluorouracil

--- Content provided by FirstRanker.com ---

O4:

Partial penectomy

Ans: 4

Ques No: 40

--- Content provided by FirstRanker.com ---


Subject:
Surgery

Topic:
Others

--- Content provided by FirstRanker.com ---

Sub-Topic:

Identify the procedure being performed in the image given below.

O1:

Intraosseou

--- Content provided by FirstRanker.com ---

s cannula for pain relief

O2:

Bone marrow aspiration

--- Content provided by FirstRanker.com ---

O3:

Intraosseous route access for giving IV fluids

O4:

Intraosseous abscess drainage

--- Content provided by FirstRanker.com ---




Ans: 3

Ques No: 41

--- Content provided by FirstRanker.com ---

Subject:
Surgery

Topic:
Urology

Sub-Topic:

--- Content provided by FirstRanker.com ---

A diabetic patient presents with sudden-onset perineal pain. On examination, foul-smel ing
discharge, and necrotic tissue as seen in the image is noted. Which of the fol owing is true
about the given condition?

O1:

--- Content provided by FirstRanker.com ---

Anti-gas gangrene serum indicated for al cases

O2:

Urinary diversion is the next step

--- Content provided by FirstRanker.com ---

O3:

Bilateral orchidectomy must be done

O4:

Mixed aerobic and anaerobic infection

--- Content provided by FirstRanker.com ---


Ans: 4

Ques No: 4 2

Subject:
Surgery

--- Content provided by FirstRanker.com ---


Topic:
Gastrointestinal Surgery

Sub-Topic:

Which of the fol owing is the most common complication fol owing ligation of the first vessel

--- Content provided by FirstRanker.com ---

during abdominoperineal resection for rectal carcinoma?
(APR Abdominoperineal resection)

O1:

Parasympathetic-bladder dysfunction and retrograde ejaculation.

--- Content provided by FirstRanker.com ---


O2:

Sympathetic - - bladder dysfunction and impotence.



--- Content provided by FirstRanker.com ---

O3:

Sympathetic- retrograde ejaculation and bladder dysfunction.

O4:

Sympathetic-Impotence and loss of cutaneous sensation in perineal region

--- Content provided by FirstRanker.com ---


Ans: 3

Ques No: 43

Subject:
Surgery

--- Content provided by FirstRanker.com ---


Topic:
Plastic Surgery

Sub-Topic:

A patient presents to the clinic with the lesion given in the image. He had a traumatic injury to

--- Content provided by FirstRanker.com ---

the chest one year ago. What is the most likely diagnosis?

O1:

Hemangioma

--- Content provided by FirstRanker.com ---

O2:

Hypertrophic scar

O3:

Keloid

--- Content provided by FirstRanker.com ---


O4:

Neurofibro ma

Ans: 3

--- Content provided by FirstRanker.com ---

Ques No: 44

Subject:
Surgery

Topic:
Gastrointestinal Surgery

--- Content provided by FirstRanker.com ---


Sub-Topic:

A 45-year-old male underwent bilateral laparoscopic hernia repair for inguinal hernia.
Postoperatively, he complained of pain in the right thigh. Which of the fol owing nerve
entrapment leads to this symptom?

--- Content provided by FirstRanker.com ---



O1:

Ilioinguinal nerve

O2:

--- Content provided by FirstRanker.com ---

Iliohypogastric nerve

O3:

Femoral nerve

--- Content provided by FirstRanker.com ---

O4:

Lateral cutaneous nerve of thigh

Ans: 4

Ques No: 45

--- Content provided by FirstRanker.com ---


Subject:
Surgery

Topic:
Others

--- Content provided by FirstRanker.com ---

Sub-Topic:

A patient was brought to the ER fol owing a road traf ic accident. On examination, the patient
opens his eyes to a painful stimulus, speaks inappropriately, and withdraws his limbs to a
painful stimulus. What is his GCS score?

--- Content provided by FirstRanker.com ---

O1:

E2V2M3

O2:

E3V3M3

--- Content provided by FirstRanker.com ---


O3:

E2V3M4

O4:

--- Content provided by FirstRanker.com ---

E3V2M2

Ans: 3

Ques No: 4 6

--- Content provided by FirstRanker.com ---

Subject:
Surgery

Topic:
Hepatobiliary Pancreatic Surgery

Sub-Topic:

--- Content provided by FirstRanker.com ---

A patient presented with fever and abdominal pain with jaundice. Investigations showed the
given findings. Which of the fol owing statements is true regarding this condition?

O1:

Surgery is mandatory

--- Content provided by FirstRanker.com ---


O2:

Fine-needle aspiration cytology is diagnostic

O3:

--- Content provided by FirstRanker.com ---

Angioembolization is the treatment of choice

O4:



Echinococcus species is involved in etiology

--- Content provided by FirstRanker.com ---


Ans: 4

Ques No: 47

Subject:
Surgery

--- Content provided by FirstRanker.com ---


Topic:
Oncology

Sub-Topic:

A patient who is a known case of alcohol dependence syndrome presents with sudden and

--- Content provided by FirstRanker.com ---

unintentional weight loss. What is the most likely diagnosis?
-ALP 240 IU/L
-S. Direct bilirubin - 1 mg/dL
-AST/ALT -0.5
-Alpha fetoprotein- 600 ng/mL

--- Content provided by FirstRanker.com ---


O1:

Hepatic adenoma

O2:

--- Content provided by FirstRanker.com ---

Cholangioca rcinoma

O3:

Hepatocel ular carcinoma

--- Content provided by FirstRanker.com ---

O4:

Alcoholic hepatitis

Ans: 3

Ques No: 48

--- Content provided by FirstRanker.com ---


Subject:
Dermatology

Topic:
Miscel aneous Disorders

--- Content provided by FirstRanker.com ---

Sub-Topic:

A lady developed a skin reaction, as shown in the image, after using a hair dye. Which of the


fol owing chemicals is responsible for this condition?

--- Content provided by FirstRanker.com ---

O1:

Pol en

O2:

Chromates

--- Content provided by FirstRanker.com ---


O3:

Balsam of Peru

O4:

--- Content provided by FirstRanker.com ---

p-Phenylen ediamine

Ans: 4

Ques No: 49

--- Content provided by FirstRanker.com ---

Subject:
Dermatology

Topic:
Introduction to Dermatology

Sub-Topic:

--- Content provided by FirstRanker.com ---

A female patient presented with acne that is not resolving on oral isotretinoin and antibiotics
therapy. Which of the fol owing is the next best investigation?

O1:

Look for dietary triggers.

--- Content provided by FirstRanker.com ---


O2:



Evaluate for hyperandrogenism

--- Content provided by FirstRanker.com ---

O3:

Check for antibiotic resistance

O4:

Look for drug triggers

--- Content provided by FirstRanker.com ---


Ans: 2

Ques No: 50

Subject:
Anaesthesia

--- Content provided by FirstRanker.com ---


Topic:
Cardiopulmonary Cerebral Resuscitation

Sub-Topic:

Identify the maneuver being performed in the given image.

--- Content provided by FirstRanker.com ---


O1:

Head tilt, chi n lift

O2:

--- Content provided by FirstRanker.com ---

Jaw thrust

O3:

Head extension

--- Content provided by FirstRanker.com ---

O4:

d. In-line manual stabilization

Ans: 1

Ques No: 51

--- Content provided by FirstRanker.com ---


Subject:
Surgery


Topic:
Hepatobiliary Pancreatic Surgery

--- Content provided by FirstRanker.com ---

Sub-Topic:

A patient presented with features of chronic pancreatitis with recurrent attacks and has a 10
mm dilatation of the pancreatic duct with intraductal calculi present. Which of the fol owing is
the surgery of choice?

--- Content provided by FirstRanker.com ---

O1:

Pancreaticoduodenectomy

O2:

Longitudinal pancreaticojejunostomy

--- Content provided by FirstRanker.com ---


O3:

ERCP and sphincterotomy

O4:

--- Content provided by FirstRanker.com ---

Coring of pancreas head

Ans: 2

Ques No: 52

--- Content provided by FirstRanker.com ---

Subject:
Dermatology

Topic:
Sexual y Transmitted Infections

Sub-Topic:

--- Content provided by FirstRanker.com ---

A young woman complains of a painless ulcer in the genital area. It is associated with
non-tender inguinal lymphadenopathy. What is the most likely diagnosis?

O1:

Chancroid

--- Content provided by FirstRanker.com ---


O2:

Syphilis

O3:

--- Content provided by FirstRanker.com ---

Herpes genitalis

O4:

Granuloma i nguinale

--- Content provided by FirstRanker.com ---

Ans: 2

Ques No: 53

Subject:
Dermatology

--- Content provided by FirstRanker.com ---

Topic:
Miscel aneous Disorders

Sub-Topic:

Which of the fol owing is associated with the clinical condition shown in the image?

--- Content provided by FirstRanker.com ---

O1:

Cataract

O2:

Glaucoma

--- Content provided by FirstRanker.com ---




O3:

Malignant melanoma

--- Content provided by FirstRanker.com ---

O4:

Basal cel carcinoma

Ans: 3

Ques No: 54

--- Content provided by FirstRanker.com ---


Subject:
Anaesthesia

Topic:
Pre-Anaesthetic Evaluation

--- Content provided by FirstRanker.com ---

Sub-Topic:

Which of the fol owing is the drug of choice for preoperative antibiotic prophylaxis in a patient
undergoing cardiac surgery?

O1:

--- Content provided by FirstRanker.com ---

Penicil in G

O2:

Erythromycin

--- Content provided by FirstRanker.com ---

O3:

Azithromyci n

O4:

Cefazolin

--- Content provided by FirstRanker.com ---


Ans: 4



Ques No: 55

--- Content provided by FirstRanker.com ---

Subject:
ENT

Topic:
Nose and Paranasal Sinuses

Sub-Topic:

--- Content provided by FirstRanker.com ---

A 30-year-old male presents with nonaxial proptosis of the left eye. The patient gives a history
of a road traf ic accident 15 years back. The CT image is given below. What is the most likely
diagnosis?


O1:

--- Content provided by FirstRanker.com ---

Frontal mucocele

O2:

Frontal meningioma

--- Content provided by FirstRanker.com ---

O3:

Juvenile nasopharyngeal angiofibroma

O4:

Pseudotumor of orbit

--- Content provided by FirstRanker.com ---


Ans: 1

Ques No: 56

Subject:
Surgery

--- Content provided by FirstRanker.com ---


Topic:
Endocrine Surgery

Sub-Topic:

A woman presented with a BIRADS-5 breast lesion. Which of the fol owing is a good

--- Content provided by FirstRanker.com ---

prognostic factor for this lesion?

O1:

BRCA-1 positive

--- Content provided by FirstRanker.com ---

O2:

p53 positive

O3:

ER positive

--- Content provided by FirstRanker.com ---


O4:

High Ki-67

Ans: 3

--- Content provided by FirstRanker.com ---

Ques No: 57

Subject:
ENT

Topic:
La rynx

--- Content provided by FirstRanker.com ---


Sub-Topic:

A 55-year-old patient comes with hoarseness of voice and dif iculty swal owing. The patient
was diagnosed with laryngeal carcinoma, and surgical management was done. The
post-operative image of the patient is given below. Which of the fol owing surgery was done on

--- Content provided by FirstRanker.com ---

this patient?



O1:

Partial laryngectomy

--- Content provided by FirstRanker.com ---


O2:

Percutaneous tracheostomy

O3:

--- Content provided by FirstRanker.com ---

Standard tracheostomy

O4:

Total laryngectomy

--- Content provided by FirstRanker.com ---

Ans: 4

Ques No: 58

Subject:
Surgery

--- Content provided by FirstRanker.com ---

Topic:
Cardiothoracic Vascular Surgery

Sub-Topic:

A patient p resents with sudden onset of chest pain shooting to the neck and interscapular
region. X-ray shows widened mediastinum. BP is 110/90 mmHg in the right upper limb and

--- Content provided by FirstRanker.com ---

160/100 mmHg in the left upper limb. What is the most likely diagnosis?

O1:

Acute coronary syndrome

--- Content provided by FirstRanker.com ---

O2:

Acute pulmonary embolism

O3:

Acute aortic dissection

--- Content provided by FirstRanker.com ---


O4:

Esophageal rupture


Ans: 3

--- Content provided by FirstRanker.com ---

Ques No: 59

Subject:
ENT

Topic:
Ear

--- Content provided by FirstRanker.com ---


Sub-Topic:

A female patient with hearing loss is examined and is found to be Rinne negative at 256 Hz
and 512 Hz, while Rinne positive at 1024 Hz. What is the expected air conduction and bone
conduction gap?

--- Content provided by FirstRanker.com ---


O1:

30-45 dB

O2:

--- Content provided by FirstRanker.com ---

15-30 dB

O3:

45-60 dB

--- Content provided by FirstRanker.com ---

O4:

>60 dB

Ans: 1

Ques No: 60

--- Content provided by FirstRanker.com ---


Subject:
Surgery

Topic:
Hepatobiliary Pancreatic Surgery

--- Content provided by FirstRanker.com ---

Sub-Topic:

A patient presented with right hypochondriac pain. He had an episode of diarrhea 1 week
prior. CT scan of the abdomen reveals a liver abscess of around 25 ccs. What is the next step
in management?

--- Content provided by FirstRanker.com ---

O1:

PAIR

O2:

Surgery

--- Content provided by FirstRanker.com ---


O3:

Medical the rapy

O4:

--- Content provided by FirstRanker.com ---

Percutaneous drainage

Ans: 3

Ques No: 61

--- Content provided by FirstRanker.com ---

Subject:
Orthopaedics

Topic:
Pediatric Orthopedics

Sub-Topic:

--- Content provided by FirstRanker.com ---

A child was brought with bony deformities, as shown in the radiograph below. It could be due


to the deficiency of which of the fol owing?

O1:

--- Content provided by FirstRanker.com ---

Vitamin A

O2:

Vitamin D

--- Content provided by FirstRanker.com ---

O3:

Vitamin K

O4:

Vitamin C

--- Content provided by FirstRanker.com ---


Ans: 2

Ques No: 62

Subject:
Surgery

--- Content provided by FirstRanker.com ---


Topic:
Neurosurgery

Sub-Topic:

A 50-year-old patient with a history of trauma one week back presents with confusion and

--- Content provided by FirstRanker.com ---



ataxia. He is on treatment for alcohol dependence and his last intake was 2 weeks back. CT
scan reveals the fol owing finding. What is the diagnosis?

O1:

--- Content provided by FirstRanker.com ---

Extradural hemorrhage

O2:

Subdural hemorrhage

--- Content provided by FirstRanker.com ---

O3:

Intra-parenchymal hemorrhage

O4:

Subarachno id hemorrhage

--- Content provided by FirstRanker.com ---


Ans: 2

Ques No: 63

Subject:
Surgery

--- Content provided by FirstRanker.com ---


Topic:
Plastic Surgery

Sub-Topic:

Which of the fol owing is false about the given condition?

--- Content provided by FirstRanker.com ---


O1:

Repeated radical surgery every 24 hours may be required to adequately manage this
condition


--- Content provided by FirstRanker.com ---


O2:

This is a case of Meleney's gangrene

O3:

--- Content provided by FirstRanker.com ---

The condition is caused by hemolytic streptococcus and may sometimes be polymicrobial

O4:

Hyperbaric oxygen has no role in the treatment of this condition

--- Content provided by FirstRanker.com ---

Ans: 4

Ques No: 64

Subject:
Pharmacology

--- Content provided by FirstRanker.com ---

Topic:
Antimicrobials : Anti Bacterial Drugs

Sub-Topic:

A bronchial asthma patient on inhalational steroids presented with white patchy lesions on the
tongue and buccal mucosa. What is the drug that can be used to treat this condition?

--- Content provided by FirstRanker.com ---


O1:

Clotrimazole

O2:

--- Content provided by FirstRanker.com ---

Griseofulvin

O3:

Terbinafine

--- Content provided by FirstRanker.com ---

O4:

Flucytosine

Ans: 1

Ques No: 65

--- Content provided by FirstRanker.com ---


Subject:
Surgery


Topic:
Gastrointestinal Surgery

--- Content provided by FirstRanker.com ---

Sub-Topic:

A patient presents with a swel ing in the groin region and it is diagnosed as a direct inguinal
hernia. Weakness in which of the fol owing structures is most likely to cause this condition?

O1:

--- Content provided by FirstRanker.com ---

Pectineal ligament

O2:

Conjoint tendon

--- Content provided by FirstRanker.com ---

O3:

Reflected part of inguinal ligament

O4:

Lacunar ligament

--- Content provided by FirstRanker.com ---


Ans: 2

Ques No: 66

Subject:
Pharmacology

--- Content provided by FirstRanker.com ---


Topic:
General Pharmacology

Sub-Topic:

A pregnant woman with a history of bronchial asthma is in the third stage of labor. Which

--- Content provided by FirstRanker.com ---

drugs should be avoided in managing postpartum hemorrhage in this mother?

O1:

Carboprost

--- Content provided by FirstRanker.com ---

O2:

Oxytocin

O3:

Dinoprostone

--- Content provided by FirstRanker.com ---


O4:

Methyl ergo metrine

Ans: 1

--- Content provided by FirstRanker.com ---

Ques No: 67

Subject:
Orthopaedics

Topic:
Peripheral Nerve Injuries

--- Content provided by FirstRanker.com ---


Sub-Topic:

In which of the fol owing nerve injuries is the instrument shown below used?



--- Content provided by FirstRanker.com ---

O1:

Radial nerve

O2:

Median nerve

--- Content provided by FirstRanker.com ---


O3:

Ulnar nerve

O4:

--- Content provided by FirstRanker.com ---

Volkmann's i schemic contracture

Ans: 1

Ques No: 68

--- Content provided by FirstRanker.com ---

Subject:
Pharmacology

Topic:
CVS Pharmacology

Sub-Topic:

--- Content provided by FirstRanker.com ---

Name the drug that acts on both the marked areas.



O1:

Sacubitril

--- Content provided by FirstRanker.com ---


O2:

Omapatrilat

O3:

--- Content provided by FirstRanker.com ---

Losartan

O4:

Nesiritide

--- Content provided by FirstRanker.com ---

Ans: 2

Ques No: 6 9

Subject:
Orthopaedics

--- Content provided by FirstRanker.com ---

Topic:
Spine + Pelvis + Lower Limb Traumatology

Sub-Topic:

A patient presented with chronic knee pain but has no history of trauma. Identify the condition
shown in the radiograph below and the appropriate management.

--- Content provided by FirstRanker.com ---




O1:

Patel ar avulsion fracture, TBW

--- Content provided by FirstRanker.com ---

O2:

Bipartite fracture patel a, X-ray of other knees

O3:

Fracture of the upper pole of the patel a, Cylindrical cast

--- Content provided by FirstRanker.com ---


O4:

Avulsion fracture, interfragmentary screw fixation

Ans: 2

--- Content provided by FirstRanker.com ---

Ques No: 70

Subject:
Surgery

Topic:
Cardiothoracic Vascular Surgery

--- Content provided by FirstRanker.com ---


Sub-Topic:

A construction worker came to the OPD complaining of white fingers. He has been working in
the cement and concrete industry and has been working on heavy machinery dril s, wood, and
furniture polishing for 20 years. What is the most likely etiology for this condition?

--- Content provided by FirstRanker.com ---


O1:

Candidal inf ection of the fingers due to continuous exposure to water

O2:

--- Content provided by FirstRanker.com ---

Exposure to thinners and paints

O3:

Continuous exposure to cement and concrete

--- Content provided by FirstRanker.com ---

O4:

Continuous exposure to dril s and machines

Ans: 4

Ques No: 71

--- Content provided by FirstRanker.com ---


Subject:
Pharmacology

Topic:
CVS Pharmacology

--- Content provided by FirstRanker.com ---

Sub-Topic:



A patient with a previous history of myocardial infarction and ventricular arrhythmia is on
treatment for a few months. He developed fatigue, dyspnea, and weight gain. He had also
developed pulmonary fibrosis. Which of the fol owing drugs is responsible for the side ef ects

--- Content provided by FirstRanker.com ---

mentioned above?

O1:

Amiodarone

--- Content provided by FirstRanker.com ---

O2:

Atenolol

O3:

Aspirin

--- Content provided by FirstRanker.com ---


O4:

Spironolactone

Ans: 1

--- Content provided by FirstRanker.com ---

Ques No: 72

Subject:
Surgery

Topic:
Gastrointestinal Surgery

--- Content provided by FirstRanker.com ---


Sub-Topic:

A 5-year-old child was admitted to the hospital for prolapsing rectal mass and painless rectal
bleeding. Histopathological examination reveals enlarged and inflamed glands fil ed with
mucin. What is the likely diagnosis?

--- Content provided by FirstRanker.com ---


O1:

Adenoma

O2:

--- Content provided by FirstRanker.com ---

Choristoma

O3:

Hamartoma

--- Content provided by FirstRanker.com ---

O4:

Carcinoma

Ans: 3

Ques No: 7 3

--- Content provided by FirstRanker.com ---


Subject:
Pharmacology

Topic:
Autacoids

--- Content provided by FirstRanker.com ---

Sub-Topic:

A young female comes to the emergency department after consuming 100 aspirin tablets.
What should be the next step in management?

O1:

--- Content provided by FirstRanker.com ---

N-acetyl cysteine to replenish glutathione stores

O2:

Pralidoxime


--- Content provided by FirstRanker.com ---


O3:

Glucagon to control bradycardia and hypoglycemia

O4:

--- Content provided by FirstRanker.com ---

Give sodium bicarbonate to alkalize urine

Ans: 4

Ques No: 74

--- Content provided by FirstRanker.com ---

Subject:
Pharmacology

Topic:
CVS Pharmacology

Sub-Topic:

--- Content provided by FirstRanker.com ---

A chronic smoker was on nicotine replacement therapy and clonidine tablets for smoking
de-addiction. He stopped taking clonidine tablets and now presents with a headache. What is
the reason behind this condition?

O1:

--- Content provided by FirstRanker.com ---

Postural hypotension

O2:

Receptor upregulation

--- Content provided by FirstRanker.com ---

O3:

Rebound hypertension

O4:

Receptor hypersensitivity

--- Content provided by FirstRanker.com ---


Ans: 3

Ques No: 75

Subject:
Surgery

--- Content provided by FirstRanker.com ---


Topic:
Plastic Surgery

Sub-Topic:

Identify the given condition.

--- Content provided by FirstRanker.com ---


O1:

Venous ulcer


O2:

--- Content provided by FirstRanker.com ---

Malignant ulcer

O3:

Arterial ulcer

--- Content provided by FirstRanker.com ---

O4:

Trophic ulcer

Ans: 4

Ques No: 76

--- Content provided by FirstRanker.com ---


Subject:
Pharmacology

Topic:
Autacoids

--- Content provided by FirstRanker.com ---

Sub-Topic:

A patient with recurrent gout attacks was started on a therapy that inhibits uric acid synthesis.
His symptoms were reduced after therapy. Which drug was he started on?

O1:

--- Content provided by FirstRanker.com ---

Probenecid

O2:

Colchicine

--- Content provided by FirstRanker.com ---

O3:

Diclofenac

O4:

Allopurinol

--- Content provided by FirstRanker.com ---


Ans: 4

Ques No: 77

Subject:
Pharmacology

--- Content provided by FirstRanker.com ---


Topic:
Drugs Affecting Blood and Blood formation

Sub-Topic:

A patient wit h a malignancy is undergoing chemotherapy. The platelet counts were reduced

--- Content provided by FirstRanker.com ---

after the previous cycle of chemotherapy. Which of the fol owing drugs can be used to treat
this patient?

O1:

Oprelvekin (IL-11)

--- Content provided by FirstRanker.com ---


O2:

Filgrastim

O3:

--- Content provided by FirstRanker.com ---

Erythropoietin

O4:

Amifostine

--- Content provided by FirstRanker.com ---

Ans: 1

Ques No: 78


Subject:
Pharmacology

--- Content provided by FirstRanker.com ---

Topic:
CVS Pharmacology

Sub-Topic:

The true statement among the fol owing is___

--- Content provided by FirstRanker.com ---

O1:

The dose of telmisartan should be reduced in renal failure but not in hepatic failure.

O2:

The dose of irbesartan should be reduced in case of mild-moderate hepatic failure and renal

--- Content provided by FirstRanker.com ---

failure.

O3:

The dose of candesartan should be reduced in mild-moderate liver failure but not in renal
failure.

--- Content provided by FirstRanker.com ---


O4:

Losartan acts as a thromboxane A2 antagonist and inhibits platelet aggregation.

Ans: 4

--- Content provided by FirstRanker.com ---

Ques No: 79

Subject:
Orthopaedics

Topic:
Peripheral Nerve Injuries

--- Content provided by FirstRanker.com ---


Sub-Topic:

A patient at the orthopedics OPD complains of troubled sleep at night due to numbness and
tingling sensation involving his lateral 3 digits. His symptoms are relieved as he lays his arms
hanging from the bed. Which of the fol owing options correctly describes his condition and the

--- Content provided by FirstRanker.com ---

test used to assess it?

O1:

Guyon's canal syndrome, Froment's test

--- Content provided by FirstRanker.com ---

O2:

Carpal tunnel syndrome, Froment's test

O3:

Guyon's can al syndrome, Durkan's test

--- Content provided by FirstRanker.com ---


O4:

Carpal tunnel syndrome, Durkan's test

Ans: 4

--- Content provided by FirstRanker.com ---

Ques No: 80

Subject:
Orthopaedics

Topic:
Metabolic Disorders

--- Content provided by FirstRanker.com ---


Sub-Topic:

A child is brought to the orthopedics OPD with a deformity in the lower limb and
hyperpigmented skin lesions. The x-ray of her thigh is shown below. What is the most likely
diagnosis?

--- Content provided by FirstRanker.com ---




O1:

Non-ossifying fibroma

--- Content provided by FirstRanker.com ---

O2:

Fibrous dysplasia

O3:

Paget's disease

--- Content provided by FirstRanker.com ---


O4:

Osteogenesis imperfecta

Ans: 2

--- Content provided by FirstRanker.com ---

Ques No: 81

Subject:
Psychiatry

Topic:
Schizophrenia Spectrum and Other Psychotic Disorders

--- Content provided by FirstRanker.com ---


Sub-Topic:

A schizoph renic patient was prescribed drug A after he did not respond to haloperidol and
thioridazine. He now presents with excessive salivation, an increase in blood glucose, and
hyperlipidemia. What is drug A?

--- Content provided by FirstRanker.com ---


O1:

Ziprasidone

O2:

--- Content provided by FirstRanker.com ---

Risperidone

O3:

Clozapine

--- Content provided by FirstRanker.com ---

O4:

Aripiprazole

Ans: 3



--- Content provided by FirstRanker.com ---

Ques No: 82

Subject:
Orthopaedics

Topic:
Orthopedics Oncology

--- Content provided by FirstRanker.com ---


Sub-Topic:

The given image is an x-ray of a 22-year-old female. What is the probable diagnosis?

O1:

--- Content provided by FirstRanker.com ---

Chondroblastoma

O2:

Osteochondroma

--- Content provided by FirstRanker.com ---

O3:

Giant cel tumor

O4:

Aneurysmal bone cyst

--- Content provided by FirstRanker.com ---


Ans: 3

Ques No: 8 3

Subject:
Physiology

--- Content provided by FirstRanker.com ---


Topic:
Respiratory System

Sub-Topic:

Identify the marked region in the given spirometry.

--- Content provided by FirstRanker.com ---




O1:

Functional residual capacity

--- Content provided by FirstRanker.com ---

O2:

Tidal volume

O3:

Expiratory reserve volume

--- Content provided by FirstRanker.com ---


O4:

Expiratory capacity

Ans: 1

--- Content provided by FirstRanker.com ---

Ques No: 8 4

Subject:
Surgery

Topic:
Cardiothoracic Vascular Surgery

--- Content provided by FirstRanker.com ---


Sub-Topic:

A patient with varicose veins complains of eczema over the same region. What is the stage of
the clinical classification according to the CEAP guidelines?
CEAP-Clinical-Aetiology-Anatomy- Pathophysiology classification of chronic venous disorders

--- Content provided by FirstRanker.com ---


O1:

C2

O2:

--- Content provided by FirstRanker.com ---

C3

O3:

C4a


--- Content provided by FirstRanker.com ---

O4:

C4b

Ans: 3

Ques No: 85

--- Content provided by FirstRanker.com ---


Subject:
Surgery

Topic:
Neurosurgery

--- Content provided by FirstRanker.com ---

Sub-Topic:

A patient comes to the casualty with a severe headache. His BP was found to be 160/100
mmHg. CT scan revealed a subarachnoid hemorrhage. What is the next best step in the
management of this patient?

--- Content provided by FirstRanker.com ---

O1:

Nimodipine

O2:

Angiography

--- Content provided by FirstRanker.com ---


O3:

Surgery

O4:

--- Content provided by FirstRanker.com ---

Fibrinolytic therapy

Ans: 2

Ques No: 86

--- Content provided by FirstRanker.com ---

Subject:
Physiology

Topic:
General Physiology

Sub-Topic:

--- Content provided by FirstRanker.com ---

A man was brought to the emergency after suddenly becoming unconscious while working in
the field. On examination, his temperature was 105 degrees Fahrenheit, and his skin turgor
was decrea sed. Which of the folowing would not be seen in the patient?

O1:

--- Content provided by FirstRanker.com ---

Tachypnea

O2:

Hypotension

--- Content provided by FirstRanker.com ---

O3:

Sweating

O4:

Red hot skin

--- Content provided by FirstRanker.com ---


Ans: 3

Ques No: 87

Subject:
Physiology

--- Content provided by FirstRanker.com ---


Topic:
Excretory System

Sub-Topic:



--- Content provided by FirstRanker.com ---

Identify the correctly matched pair of substances with their renal clearance from the graph
below.

O1:

A-Glucose, B-PAH, C- Bicarbonate and D - Inulin

--- Content provided by FirstRanker.com ---


O2:

A-Glucose, B- Bicarbonate, C-Inulin and D - PAH

O3:

--- Content provided by FirstRanker.com ---

A-PAH, B-Inulin, C - Glucose and D - Bicarbonate

O4:

A - Inulin, B - Glucose, C - Bicarbonate and D - PAH

--- Content provided by FirstRanker.com ---

Ans: 2

Ques No: 8 8

Subject:
Surgery

--- Content provided by FirstRanker.com ---

Topic:
Gastrointestinal Surgery

Sub-Topic:

A 20-year-old female patient consumed a cleaning product containing 90% sodium hydroxide.
She is having complete dysphagia. What is the best step in management?

--- Content provided by FirstRanker.com ---


O1:

Esophagojejunostomy

O2:

--- Content provided by FirstRanker.com ---

Stent placement

O3:



Feeding jejunostomy

--- Content provided by FirstRanker.com ---


O4:

Gastrojejunostomy

Ans: 3

--- Content provided by FirstRanker.com ---

Ques No: 89

Subject:
Surgery

Topic:
Endocrine Surgery

--- Content provided by FirstRanker.com ---


Sub-Topic:

A 23-year-old male patient presents with midline swel ing in the neck. The swel ing moves with
deglutition and protrusion of the tongue. What is the likely diagnosis?

--- Content provided by FirstRanker.com ---

O1:

Brachial cyst

O2:

Thyroglossal cyst

--- Content provided by FirstRanker.com ---


O3:

Plunging ranula

O4:

--- Content provided by FirstRanker.com ---

Dermoid cyst

Ans: 2

Ques No: 90

--- Content provided by FirstRanker.com ---

Subject:
Physiology

Topic:
Nerve Muscle Physiology

Sub-Topic:

--- Content provided by FirstRanker.com ---

Identify the true statement regarding the given nerve action potential curve?



O1:

The threshol d point is at A

--- Content provided by FirstRanker.com ---


O2:

At point E, the nerve is more excitable

O3:

--- Content provided by FirstRanker.com ---

Point C to D is due to the opening of Na+ and closure of K+ channels

O4:

Point B to D is a refractory period

--- Content provided by FirstRanker.com ---

Ans: 4

Ques No: 91

Subject:
Surgery

--- Content provided by FirstRanker.com ---

Topic:
Others


Sub-Topic:

A young patient is admitted with a history of fever for 5 days. BP is 90/60 mmHg, PR-120 bpm,
RR-24 breaths/ min, GCS score-10; laboratory tests show leucocytosis with neutrophilia and

--- Content provided by FirstRanker.com ---

serum creatinine level of 2.6 mg/ dL. An intern doctor wants to calculate the qSOFA score to
predict prognosis and hospitalization duration. Which of the fol owing comprises the qSOFA
score?

O1:

--- Content provided by FirstRanker.com ---

BP, RR and CBC

O2:

Creatinine, PR, BP

--- Content provided by FirstRanker.com ---

O3:

RR, Body temperature, PR

O4:

SBP, RR, GCS score

--- Content provided by FirstRanker.com ---


Ans: 4

Ques No: 92

Subject:
Physiology

--- Content provided by FirstRanker.com ---


Topic:
General Physiology

Sub-Topic:

Identify the type of transport across the cel membrane given below?

--- Content provided by FirstRanker.com ---


O1:

Simple dif usion



--- Content provided by FirstRanker.com ---

O2:

Facilitated dif usion

O3:

Primary active transport

--- Content provided by FirstRanker.com ---


O4:

Secondary active transport

Ans: 2

--- Content provided by FirstRanker.com ---

Ques No: 93

Subject:
Medicine

Topic:
Cardiology

--- Content provided by FirstRanker.com ---


Sub-Topic:

Identify the condition associated with the murmur in the image below.

O1:

--- Content provided by FirstRanker.com ---

Mitral regurgitation

O2:

Aortic regurgitation

--- Content provided by FirstRanker.com ---

O3:

Aortic stenosis



O4:

--- Content provided by FirstRanker.com ---

Mitral stenosis

Ans: 1

Ques No: 94

--- Content provided by FirstRanker.com ---

Subject:
Psychiatry

Topic:
Mood Disorders

Sub-Topic:

--- Content provided by FirstRanker.com ---

A woman, w ho is 4 days postpartum, presented with tearfulness, mood swings, and
occasional insomnia. What is the likely diagnosis?

O1:

Postpartum depression

--- Content provided by FirstRanker.com ---


O2:

Postpartum blues

O3:

--- Content provided by FirstRanker.com ---

Postpartum psychosis

O4:

Postpartum anxiety

--- Content provided by FirstRanker.com ---

Ans: 2

Ques No: 95


Subject:
Psychiatry

--- Content provided by FirstRanker.com ---

Topic:
Neurotic, Stress-Related and Somatoform Disorders

Sub-Topic:

A 25-year-old female reported symptoms of anxiety, palpitations, sweating, breathlessness,
chest pain, and a feeling of impending doom. She recal s having experienced 5-6 similar

--- Content provided by FirstRanker.com ---

episodes in the past 6 months, each episode lasting for about 20-30 minutes. What is the
likely diagnosis?

O1:

Depression

--- Content provided by FirstRanker.com ---


O2:

Panic disorder

O3:

--- Content provided by FirstRanker.com ---

Generalized anxiety disorder

O4:

Phobia

--- Content provided by FirstRanker.com ---

Ans: 2

Ques No: 96

Subject:
Medicine

--- Content provided by FirstRanker.com ---

Topic:
Rheumatology / Connective Tissue Disorder

Sub-Topic:

A young man came to the medical OPD with complaints of early morning backache and
stif ness, which improves on exercise, and persistent red eyes. On examination, lung

--- Content provided by FirstRanker.com ---

expansion was less than 3 cm. X-ray is shown in the image given below.


What is the most probable diagnosis?

O1:

--- Content provided by FirstRanker.com ---

Ankylosing spondylitis

O2:

Paget's disease

--- Content provided by FirstRanker.com ---

O3:

Healed tube rculosis

O4:

Osteopetrosis

--- Content provided by FirstRanker.com ---


Ans: 1

Ques No: 97

Subject:
Medicine

--- Content provided by FirstRanker.com ---


Topic:
Nephrology / Kidney Disease

Sub-Topic:

A patient with hyperkalemia and elevated urea levels underwent dialysis. Towards the end of

--- Content provided by FirstRanker.com ---

the session, she became drowsy and had a sudden seizure episode. On examination, the
patient was hypotensive. What is the treatment for this condition?

O1:



--- Content provided by FirstRanker.com ---

Bumetanide

O2:

Ethacrynic acid

--- Content provided by FirstRanker.com ---

O3:

Nesiritide

O4:

IV Mannitol

--- Content provided by FirstRanker.com ---


Ans: 4

Ques No: 98

Subject:
Medicine

--- Content provided by FirstRanker.com ---


Topic:
Endocrinology

Sub-Topic:

A female patient presents to the emergency department with severe restlessness, palpitations,

--- Content provided by FirstRanker.com ---

and tremors. She is a known case of bronchial asthma. On examination, the neck looks
swol en. Blood pressure is elevated, and tachycardia is noted. ECG shows atrial fibril ation.
Which of the fol owing drugs is used for immediate management in this patient?

O1:

--- Content provided by FirstRanker.com ---

Diltiazem

O2:

Propranolol

--- Content provided by FirstRanker.com ---

O3:

Esmolol

O4:

Propylthiouracil

--- Content provided by FirstRanker.com ---


Ans: 1

Ques No: 99

Subject:
Ph ysiology

--- Content provided by FirstRanker.com ---


Topic:
Respiratory System

Sub-Topic:

A preterm baby who was delivered at 28 weeks developed respiratory distress syndrome.

--- Content provided by FirstRanker.com ---

Which of the fol owing is true about surface tension and compliance in this baby?

O1:

Surface tension - decreased; Compliance - increased

--- Content provided by FirstRanker.com ---

O2:

Surface tension - increased; Compliance - decreased

O3:

Both surface tension and compliance decreased

--- Content provided by FirstRanker.com ---


O4:

Both surface tension and compliance increased


Ans: 2

--- Content provided by FirstRanker.com ---

Ques No: 100

Subject:
Psychiatry

Topic:
Sexual Disorders

--- Content provided by FirstRanker.com ---


Sub-Topic:

A middle-aged man complains of very early ejaculation during intercourse. What is the
non-pharmacological management that can be advised to the patient?

--- Content provided by FirstRanker.com ---

O1:

Cognitive behavioral therapy

O2:

Exposure and response prevention therapy

--- Content provided by FirstRanker.com ---


O3:

Squeeze technique

O4:

--- Content provided by FirstRanker.com ---

Sensate focus therapy

Ans: 3

Ques No: 101

--- Content provided by FirstRanker.com ---

Subject:
Medicine

Topic:
Liver

Sub-Topic:

--- Content provided by FirstRanker.com ---

A 30-year-old male is found to be positive for HBsAg and HBeAg and is diagnosed with
chronic hepatitis B. The patient's viral load was 2x 100000 and SGPT is found to be doubled.
What is the appropriate management in this patient?
SGPT-Serum glutamate pyruvate transaminase

--- Content provided by FirstRanker.com ---

O1:

Lamivudine for 30+ weeks

O2:

Tenofovir fo r> 40 weeks

--- Content provided by FirstRanker.com ---


O3:

Pegylated interferon for 52 weeks

O4:

--- Content provided by FirstRanker.com ---

Combined pegylated interferon with lamivudine

Ans: 2

Ques No: 102

--- Content provided by FirstRanker.com ---

Subject:
Medicine

Topic:
Endocrinology

Sub-Topic:

--- Content provided by FirstRanker.com ---

A 25-year-old male patient complained of palpitations, sweating, and restlessness. He has a
complaint of sweaty palms. Clinical findings are depicted in the image given below.


What is the diagnostic test done on this patient?

--- Content provided by FirstRanker.com ---

O1:

Anti-thyroglobulin antibody

O2:

Anti-thyroid peroxidase antibody

--- Content provided by FirstRanker.com ---


O3:

Thyroid receptor antibody

O4:

--- Content provided by FirstRanker.com ---

Elevated ultrasensitive thyrotropin levels

Ans: 3

Ques No: 103

--- Content provided by FirstRanker.com ---

Subject:
Medicine

Topic:
Oncology

Sub-Topic:

--- Content provided by FirstRanker.com ---

A 50-year-old HIV patient presented with a painless lesion, as shown in the image.


What is the most likely diagnosis?

O1:

--- Content provided by FirstRanker.com ---

Basal cel carcinoma

O2:

Kaposi sarcoma

--- Content provided by FirstRanker.com ---

O3:

Malignant melanoma

O4:

Squamous cel carcinoma

--- Content provided by FirstRanker.com ---


Ans: 2

Ques No: 104

Subject:
Physiology

--- Content provided by FirstRanker.com ---


Topic:
Nerve Muscle Physiology

Sub-Topic:

A patient came to the hospital after a road traf ic accident. He had severe muscle injury, and

--- Content provided by FirstRanker.com ---

his serum K+ level was found to be 5.5 mEq/L. What wil happen to the resting membrane
potential in this patient?

O1:

No change

--- Content provided by FirstRanker.com ---


O2:



Becomes more negative

--- Content provided by FirstRanker.com ---

O3:

Becomes more positive

O4:

First becomes more positive then negative

--- Content provided by FirstRanker.com ---


Ans: 3

Ques No: 105

Subject:
Pediatrics

--- Content provided by FirstRanker.com ---


Topic:
Pediatric Respiratory Disorders

Sub-Topic:

A newborn presented with chest retractions, dyspnea, and lethargy. The pediatrician

--- Content provided by FirstRanker.com ---

diagnosed the baby with respiratory distress syndrome. This occurs due to the deficiency of:

O1:

Dipalmitoyl inositol

--- Content provided by FirstRanker.com ---

O2:

Lecithin

O3:

Sphingomyelin

--- Content provided by FirstRanker.com ---


O4:

Dipalmitoylphosphatidylethanolamine

Ans: 2

--- Content provided by FirstRanker.com ---

Ques No: 106

Subject:
Medicine

Topic:
Neurology

--- Content provided by FirstRanker.com ---


Sub-Topic:

A patient is brought to the OPD by his wife, complaining about dif iculty expressing emotions
and not parti cipating in daily activities. On examination, resting tremors and rigidity are noted.
Given the possible diagnosis, which part of the brain is af ected in this patient?

--- Content provided by FirstRanker.com ---


O1:

Basal ganglia

O2:

--- Content provided by FirstRanker.com ---

Hippocampus

O3:

Cerebel um

--- Content provided by FirstRanker.com ---

O4:

Premotor cortex

Ans: 1

Ques No: 107

--- Content provided by FirstRanker.com ---


Subject:
Medicine


Topic:
Infections

--- Content provided by FirstRanker.com ---

Sub-Topic:

A 45-year-old HIV-positive male complained of persistent cough and weight loss. He has skin
lesions that appear as umbilicated papules and nodules predominantly on the face, trunk, and
upper extremities. Chest x-ray showed multiple bilateral nodular infiltrates. His sputum
CBNAAT for tuberculosis was negative, and he has a low CD4 count. What is the probable

--- Content provided by FirstRanker.com ---

diagnosis?

O1:

HIV with disseminated histoplasmosis

--- Content provided by FirstRanker.com ---

O2:

HIV with disseminated cryptococcosis

O3:

HIV with mol uscum contagiosum

--- Content provided by FirstRanker.com ---


O4:

HIV with tuberculosis

Ans: 1

--- Content provided by FirstRanker.com ---

Ques No: 108

Subject:
Physiology

Topic:
The Nervous System

--- Content provided by FirstRanker.com ---


Sub-Topic:

A patient met with a road traf ic accident and developed a cervical spine injury. The fracture
fragment had pierced the lateral aspect of the dorsal column tract. Which of the fol owing
findings is seen in this patient?

--- Content provided by FirstRanker.com ---


O1:

Absence of ipsilateral lower limb proprioception

O2:

--- Content provided by FirstRanker.com ---

Absence of fine motor movement of fingers

O3:

Absence of i psilateral arm proprioception

--- Content provided by FirstRanker.com ---

O4:

Absence of contralateral lower limb proprioception

Ans: 3

Ques No: 109

--- Content provided by FirstRanker.com ---


Subject:
Medicine

Topic:
Neurology

--- Content provided by FirstRanker.com ---

Sub-Topic:

A male patient presented to the emergency room with seizures. He has a history of fever,
headache, and confusion. An MRI brain was done, and it showed inflammation involving the
bitemporal lobe. What is the most likely aetiology for this presentation?


--- Content provided by FirstRanker.com ---

O1:

Cytomegalovirus

O2:

Toxoplasma gondi

--- Content provided by FirstRanker.com ---


O3:

Herpes simplex virus

O4:

--- Content provided by FirstRanker.com ---

Mycobacterium tuberculosis

Ans: 3

Ques No: 110

--- Content provided by FirstRanker.com ---

Subject:
Physiology

Topic:
Cardiovascular System

Sub-Topic:

--- Content provided by FirstRanker.com ---

Identify the true statement regarding the point marked(Red) on the myocardial action potential
curve?

O1:

Due to the opening of Na+ and closure of fast K+ channels

--- Content provided by FirstRanker.com ---


O2:

Due to the slow but prolonged opening of Ca2+ channels

O3:

--- Content provided by FirstRanker.com ---

Due to the closure of Na+ and opening of the slow K+ channel

O4:

Due to Na+/K+ ATPase


--- Content provided by FirstRanker.com ---


Ans: 2

Ques No: 111

Subject:
Medicine

--- Content provided by FirstRanker.com ---


Topic:
Cardiology

Sub-Topic:

A male patient presents with sensory loss and weakness of limbs for 3 months. He also has

--- Content provided by FirstRanker.com ---

angular stomatitis. On examination, there is loss of proprioception, vibration sensations, UMN
type of lower limb weakness, and absent ankle reflex. What is the most probable diagnosis?

O1:

Extradural cord compression

--- Content provided by FirstRanker.com ---


O2:

Amyotrophic lateral sclerosis

O3:

--- Content provided by FirstRanker.com ---

Multiple sclerosis

O4:

Subacute co mbined degeneration of cord

--- Content provided by FirstRanker.com ---

Ans: 4

Ques No: 112

Subject:
Pediatrics

--- Content provided by FirstRanker.com ---

Topic:
Fluid and Electrolyte Disturbances

Sub-Topic:

A 10-year-old child weighing 30 kg presents with a history of loose stools for 2 days. On
examination, there is severe dehydration. Laboratory investigations are as fol ows. What is the

--- Content provided by FirstRanker.com ---

initial management as per ISPAD guidelines?
RBS
550mg/dL
pH
7.01

--- Content provided by FirstRanker.com ---



Na+
158mEq/L
Urine glucose
3+

--- Content provided by FirstRanker.com ---


O1:

Manage ABC, NS 20 mL/kg and start insulin after 1 hour

O2:

--- Content provided by FirstRanker.com ---

Manage ABC, NS 20 mL/kg along with insulin 0.1 IU/kg/hr

O3:

Manage ABC, NS 10 mL/kg along with insulin 0.1 IU/kg/hr

--- Content provided by FirstRanker.com ---

O4:

Manage ABC, NS 10 mL/kg and start insulin after 1 hour

Ans: 1

Ques No: 113

--- Content provided by FirstRanker.com ---


Subject:
Pediatrics

Topic:
Growth

--- Content provided by FirstRanker.com ---

Sub-Topic:

Which of the fol owing is the best sign to indicate adequate growth in an infant with a birth
weight of 2.8 kg?

O1:

--- Content provided by FirstRanker.com ---

Increase in length of 25 centimetres in the first year

O2:

Weight gain of 300 grams per month til 1 year

--- Content provided by FirstRanker.com ---

O3:

Anterior fontanel e closure by 6 months of age

O4:

Weight under the 75th percentile and height under the 25th percentile

--- Content provided by FirstRanker.com ---


Ans: 1

Ques No: 11 4

Subject:
Medicine

--- Content provided by FirstRanker.com ---


Topic:
Cardiology

Sub-Topic:

A 40-year-old female patient came with complaints of chest pain, palpitation, and shortness of

--- Content provided by FirstRanker.com ---

breath. On examination, a mid-diastolic murmur was heard, and a prominent 'a' wave was
found on JVP. What is the most likely diagnosis?

O1:

Mitral stenosis

--- Content provided by FirstRanker.com ---


O2:

Tricuspid stenosis

O3:

--- Content provided by FirstRanker.com ---



Mitral regurgitation

O4:

Tricuspid regurgitation

--- Content provided by FirstRanker.com ---


Ans: 2

Ques No: 115

Subject:
Pediatrics

--- Content provided by FirstRanker.com ---


Topic:
Infectious Diseases

Sub-Topic:

A 3-month-old baby complains of deafness, cataract, and patent ductus arteriosus. Which of

--- Content provided by FirstRanker.com ---

the fol owing is the most likely diagnosis?

O1:

Congenital herpes simplex virus infection

--- Content provided by FirstRanker.com ---

O2:

Congenital toxoplasmosis

O3:

Congenital cytomegalovirus infection

--- Content provided by FirstRanker.com ---


O4:

Congenital rubel a syndrome

Ans: 4

--- Content provided by FirstRanker.com ---

Ques No: 116

Subject:
PSM

Topic:
Nutrition and Health

--- Content provided by FirstRanker.com ---


Sub-Topic:

In a vil age, it is observed that several farmers have crossed gait and use a stick for support to
stand up and walk. Due to poor yield from farms, they consume meals containing rice and
pulses only. Supplementing their diet with which of the fol owing vitamins could have

--- Content provided by FirstRanker.com ---

prevented th is?

O1:

Vitamin A

--- Content provided by FirstRanker.com ---

O2:

Vitamin D

O3:

Vitamin C

--- Content provided by FirstRanker.com ---


O4:

Vitamin B

Ans: 3

--- Content provided by FirstRanker.com ---

Ques No: 117

Subject:
PSM

Topic:
Epidemiology

--- Content provided by FirstRanker.com ---



Sub-Topic:

Research is being conducted to find the association between aniline dye exposure and
bladder cancer in workers who have worked in the industry for >20 years. Two groups were
formed: one directly involved with dye handling and the other group consisting of of ice clerks

--- Content provided by FirstRanker.com ---

not directly exposed to the dye. Years of occupation were noted from records. What type of
study is being performed?

O1:

Retrospective cohort study

--- Content provided by FirstRanker.com ---


O2:

Prospective cohort study

O3:

--- Content provided by FirstRanker.com ---

Case-control study

O4:

Intervention and response

--- Content provided by FirstRanker.com ---

Ans: 1

Ques No: 118

Subject:
Pediatrics

--- Content provided by FirstRanker.com ---

Topic:
Pediatric Respiratory Disorders

Sub-Topic:

Chloride level in sweat is used in the diagnosis of which disease?

--- Content provided by FirstRanker.com ---

O1:

Phenylketonuria

O2:

Cystic fibrosis

--- Content provided by FirstRanker.com ---


O3:

Gaucher's disease

O4:

--- Content provided by FirstRanker.com ---

Osteogene sis imperfecta

Ans: 2

Ques No: 119

--- Content provided by FirstRanker.com ---

Subject:
Pediatrics

Topic:
Infectious Diseases

Sub-Topic:

--- Content provided by FirstRanker.com ---

A child presents with fever and a rash spreading from the face, behind cheeks, and buccal
mucosa to other body parts. On examination, Koplik's spot is present. What is the likely
diagnosis?

O1:

--- Content provided by FirstRanker.com ---

Measles

O2:

Rubel a


--- Content provided by FirstRanker.com ---

O3:

Varicel a

O4:

Mumps

--- Content provided by FirstRanker.com ---


Ans: 1

Ques No: 120

Subject:
PSM

--- Content provided by FirstRanker.com ---


Topic:
Concept of Health and Disease

Sub-Topic:

The blood pressure of a population was tracked from childhood to adulthood. It was observed

--- Content provided by FirstRanker.com ---

that those who had lower BP in childhood had low BP in adulthood, while those who had
higher BP in childhood had high BP in adulthood. This can be best described as

O1:

Rule of halves

--- Content provided by FirstRanker.com ---


O2:

Tracking of blood pressure

O3:

--- Content provided by FirstRanker.com ---

STEPwise approach

O4:

Primordial approach

--- Content provided by FirstRanker.com ---

Ans: 2

Ques No: 121

Subject:
Pediatrics

--- Content provided by FirstRanker.com ---

Topic:
Infectious Diseases

Sub-Topic:

A 3-week-old infant presents with a cough and sore throat. The mother reports that the infant
develops a paroxysm of cough folowed by apnea. The total leucocyte count is >50,000

--- Content provided by FirstRanker.com ---

cel s/L. Which of the fol owing drugs wil you prescribe for this patient?

O1:

Azithromycin

--- Content provided by FirstRanker.com ---

O2:

Amoxicil in

O3:

Cotrimoxazole

--- Content provided by FirstRanker.com ---


O4:

Clarithromycin

Ans: 1

--- Content provided by FirstRanker.com ---

Ques No: 122

Subject:
PSM



--- Content provided by FirstRanker.com ---

Topic:
Health Care of The Community in India

Sub-Topic:

A 30-week primigravida complaints of reduced vision at night. She has been avoiding papaya,
mango, and other fruits throughout her pregnancy as she thinks they could be abortifacients. It

--- Content provided by FirstRanker.com ---

is the primary duty of which of the fol owing workers to provide counseling and information to
the patient?

O1:

ANM

--- Content provided by FirstRanker.com ---


O2:

AWW

O3:

--- Content provided by FirstRanker.com ---

Trained birth attendant

O4:

ASHA

--- Content provided by FirstRanker.com ---

Ans: 4

Ques No: 123

Subject:
ENT

--- Content provided by FirstRanker.com ---

Topic:
Pharynx

Sub-Topic:

Identify the structure marked in the image.

--- Content provided by FirstRanker.com ---

O1:

Fossa of Ro
senmul er

O2:

--- Content provided by FirstRanker.com ---

Tubal tonsil

O3:

Opening of the Eustachian tube

--- Content provided by FirstRanker.com ---

O4:

Adenoid


Ans: 1

Ques No: 124

--- Content provided by FirstRanker.com ---


Subject:
PSM

Topic:
Communicable and Non-communicable Diseases

--- Content provided by FirstRanker.com ---

Sub-Topic:

A 22-year-old female comes to the STI clinic with minimal vaginal discharge. On speculum
examination, erosions are seen on the cervix. Which of the fol owing kit should be given to this
patient?

--- Content provided by FirstRanker.com ---

O1:

Green

O2:

Red

--- Content provided by FirstRanker.com ---


O3:

Grey

O4:

--- Content provided by FirstRanker.com ---

Yel ow

Ans: 3

Ques No: 125

--- Content provided by FirstRanker.com ---

Subject:
PSM

Topic:
Concept of Health and Disease

Sub-Topic:

--- Content provided by FirstRanker.com ---

The years of potential life lost could be attributed to,

O1:

Years lost to morbidity

--- Content provided by FirstRanker.com ---

O2:

Years lost due to premature death

O3:

Years lost to disability

--- Content provided by FirstRanker.com ---


O4:

Years lost to poor quality of life

Ans: 2

--- Content provided by FirstRanker.com ---

Ques No: 126

Subject:
ENT

Topic:
Pharynx

--- Content provided by FirstRanker.com ---


Sub-Topic:

The instrument shown in the image is not used for which of the fol owing?



--- Content provided by FirstRanker.com ---

O1:

Airway toileting

O2:

Upper airway examination

--- Content provided by FirstRanker.com ---


O3:

Acute nasopharyngeal obstruction

O4:

--- Content provided by FirstRanker.com ---

Prolonged mechanical ventilation

Ans: 2

Ques No: 127

--- Content provided by FirstRanker.com ---

Subject:
PSM

Topic:
Vaccines and Cold Chain

Sub-Topic:

--- Content provided by FirstRanker.com ---

In a 10-year-old school child, which of the fol owing vaccines is given as a part of the school
immunizatio n program?

O1:

Measles vaccine

--- Content provided by FirstRanker.com ---


O2:

Rotavirus vaccine

O3:

--- Content provided by FirstRanker.com ---

TT/Td vaccine

O4:

Hepatitis B vaccine

--- Content provided by FirstRanker.com ---

Ans: 3

Ques No: 128

Subject:
ENT

--- Content provided by FirstRanker.com ---

Topic:
Pharynx


Sub-Topic:

A 10-year-old child presents with throat pain, fever, and ear pain. He is diagnosed with
recurrent tonsil itis. Which nerve is responsible for the ear pain in this patient?

--- Content provided by FirstRanker.com ---


O1:

Tympanic branch of the glossopharyngeal nerve

O2:

--- Content provided by FirstRanker.com ---

Greater auricular nerve

O3:

Auriculotemporal nerve

--- Content provided by FirstRanker.com ---

O4:

Auricular branch of the vagus nerve

Ans: 1

Ques No: 129

--- Content provided by FirstRanker.com ---


Subject:
PSM

Topic:
International Health

--- Content provided by FirstRanker.com ---

Sub-Topic:

Which of the fol owing statements is true about cancer treatment according to the Colombo
plan?

O1:

--- Content provided by FirstRanker.com ---

Help with PET scan units for diagnosis of cancer

O2:

Human resource strengthening

--- Content provided by FirstRanker.com ---

O3:

Setting up chemotherapy units

O4:

Setting up cobalt therapy units

--- Content provided by FirstRanker.com ---


Ans: 4

Ques No: 1 30

Subject:
PSM

--- Content provided by FirstRanker.com ---


Topic:
Nutrition and Health

Sub-Topic:

A poor farmer with a history of successive crop failure develops progressive spastic

--- Content provided by FirstRanker.com ---

paraparesis, signs of upper motor neuron paralysis, and gait instability. Name the toxin
responsible for this condition.

O1:

Aflatoxin

--- Content provided by FirstRanker.com ---


O2:

Beta-oxalyl-amino-alanine

O3:

--- Content provided by FirstRanker.com ---

Ergot alkaloids


O4:

Fusarium toxin

--- Content provided by FirstRanker.com ---

Ans: 2

Ques No: 131

Subject:
PSM

--- Content provided by FirstRanker.com ---

Topic:
Vaccines and Cold Chain

Sub-Topic:

A young male came to the hospital with a clean-cut wound without any bleeding. The patient
received a ful course of tetanus vaccination 10 years ago. What is the best management for

--- Content provided by FirstRanker.com ---

this patient?

O1:

Human tetanus immunoglobulin and ful course of vaccine

--- Content provided by FirstRanker.com ---

O2:

Human tetanus immunoglobulin only

O3:

Single-dose tetanus toxoid

--- Content provided by FirstRanker.com ---


O4:

Ful course tetanus toxoid

Ans: 3

--- Content provided by FirstRanker.com ---

Ques No: 132

Subject:
PSM

Topic:
Environment and Health

--- Content provided by FirstRanker.com ---


Sub-Topic:

How is water col ected for bacteriological examination during a disease outbreak?

O1:

--- Content provided by FirstRanker.com ---

Col ect water from already leaking taps

O2:

Before col e cting, let water flow for at least 1 minute

--- Content provided by FirstRanker.com ---

O3:

Water sample container is kept close to the tap avoid spil age

O4:

Col ect from a gentle stream of water to avoid splashing

--- Content provided by FirstRanker.com ---


Ans: 4

Ques No: 133

Subject:
PSM

--- Content provided by FirstRanker.com ---


Topic:
Communicable and Non-communicable Diseases

Sub-Topic:

A boys' hostel has an outbreak of fever cases with headache, followed by the development of

--- Content provided by FirstRanker.com ---

pleomorphic rashes sparing palms and soles. What is the next best step in the management of


suspected cases?
VZIG-Varicel a zoster immunoglobulin

--- Content provided by FirstRanker.com ---

O1:

Isolate for 6 days after giving acyclovir, fol owed by VZIG within 72 hours of exposure.

O2:

Isolate for 12 days after giving acyclovir, fol owed by VZIG within 48 hours of exposure

--- Content provided by FirstRanker.com ---


O3:

Isolate for 6 days

O4:

--- Content provided by FirstRanker.com ---

Only give VZIG

Ans: 3

Ques No: 134

--- Content provided by FirstRanker.com ---

Subject:
Medicine

Topic:
Neurology

Sub-Topic:

--- Content provided by FirstRanker.com ---

A patient who is a known case of hypertension on multiple anti-hypertensive medications
came to OPD. His ECG finding is given below. Which of the fol owing drugs is responsible for
the ECG finding?

O1:

--- Content provided by FirstRanker.com ---

Prazosin

O2:

Metoprolol

--- Content provided by FirstRanker.com ---

O3:

Hydrochlorothiazide

O4:

Spironolactone

--- Content provided by FirstRanker.com ---


Ans: 4



Ques No: 135

--- Content provided by FirstRanker.com ---

Subject:
Medicine

Topic:
Neurology

Sub-Topic:

--- Content provided by FirstRanker.com ---

A 78-year-old woman presents with a progressive decline in daily activity. She gives a history
of convulsions and visual hal ucinations. She does not talk to anyone and keeps looking at the
sky. Pathological examination shows the presence of Lewy bodies within the neurons. What is
the most probable diagnosis?

--- Content provided by FirstRanker.com ---

O1:

Parkinson's disease

O2:

Prion disease

--- Content provided by FirstRanker.com ---


O3:

Huntington's chorea

O4:

--- Content provided by FirstRanker.com ---

Alzheimer's disease

Ans: 1

Ques No: 136

--- Content provided by FirstRanker.com ---

Subject:
Medicine

Topic:
Neurology

Sub-Topic:

--- Content provided by FirstRanker.com ---

A patient came to the medical OPD with complaints of polyuria. He has a history of undergoing
total hypophysectomy. His Na+ levels are found to be 155 mEq/ L, urine osmolarity was 200
mOsm/L. What is the definitive management in this patient?

O1:

--- Content provided by FirstRanker.com ---

DDAVP for 2 weeks and then discontinue

O2:

DDAVP sup plementation for lifelong

--- Content provided by FirstRanker.com ---

O3:

Upsetting of receptors so no treatment is required

O4:

Thiazides for 2 weeks

--- Content provided by FirstRanker.com ---


Ans: 2

Ques No: 137

Subject:
Medicine

--- Content provided by FirstRanker.com ---


Topic:
Pulmonology

Sub-Topic:

A 65-year-old chronic smoker came to the medicine outpatient department with complaints of

--- Content provided by FirstRanker.com ---



upper chest discomfort and drooping of an eyelid. He also complained of pain radiating to the
upper arm and a tingling sensation in the 4th and 5th digits of his left hand. The chest X-ray is
given below.
Which of the fol owing is the most likely diagnosis?

--- Content provided by FirstRanker.com ---


O1:

Pancoast tumour

O2:

--- Content provided by FirstRanker.com ---

Upper lobe pneumonia

O3:

Superior ve na cava obstruction

--- Content provided by FirstRanker.com ---

O4:

Aspergil oma

Ans: 1

Ques No: 138

--- Content provided by FirstRanker.com ---


Subject:
Medicine

Topic:
Rheumatology / Connective Tissue Disorder

--- Content provided by FirstRanker.com ---

Sub-Topic:

A chronic alcoholic patient presents with acute pain and swelling of the left great toe. There is
no history of trauma. Synovial fluid analysis shows raised leukocytes. Lab investigations show
normal serum uric acid levels. What is the most likely diagnosis?


--- Content provided by FirstRanker.com ---

O1:

Pseudogout

O2:

Acute gout

--- Content provided by FirstRanker.com ---


O3:

Reactive arthritis

O4:

--- Content provided by FirstRanker.com ---

Septic arthritis

Ans: 2

Ques No: 139

--- Content provided by FirstRanker.com ---

Subject:
Psychiatry

Topic:
Substance-Related and Addictive Disorders

Sub-Topic:

--- Content provided by FirstRanker.com ---

A chronic alcoholic is brought to the emergency department with confusion, ataxia, and painful
eye movements. The 6th cranial nerve is also involved. What is the likely diagnosis?

O1:

Wernicke's encephalopathy

--- Content provided by FirstRanker.com ---


O2:

Korsakof psychosis

O3:

--- Content provided by FirstRanker.com ---

Delirium tremens

O4:

De Clerambault syndrome

--- Content provided by FirstRanker.com ---

Ans: 1

Ques No: 140

Subject:
Pharmacology

--- Content provided by FirstRanker.com ---

Topic:
General Pharmacology

Sub-Topic:

A patient gi ven digoxin started having side efects like nausea and vomiting. The serum
concentration of digoxin was 4 mg/dL. The plasma therapeutic range is 1 mg/dL. If the half-life

--- Content provided by FirstRanker.com ---

of digoxin is 40 hours, how long should one wait before resuming the treatment?

O1:

40 hours

--- Content provided by FirstRanker.com ---

O2:

80 hours

O3:

120 hours

--- Content provided by FirstRanker.com ---


O4:

140-180 hours

Ans: 2

--- Content provided by FirstRanker.com ---



Ques No: 141

Subject:
Pharmacology

--- Content provided by FirstRanker.com ---

Topic:
ANS Pharmacology

Sub-Topic:

A patient comes to the casualty with organophosphate poisoning. He was started on atropine
infusion and pralidoxime. After 2 hours, the patient had a sudden rise in temperature. What is

--- Content provided by FirstRanker.com ---

the likely cause of fever?

O1:

Atropine toxicity

--- Content provided by FirstRanker.com ---

O2:

A side ef ect of pralidoxime

O3:

Due to organophosphate poisoning

--- Content provided by FirstRanker.com ---


O4:

Idiopathic

Ans: 1

--- Content provided by FirstRanker.com ---

Ques No: 142

Subject:
Gynaecology & Obstetrics

Topic:
Obstetrics

--- Content provided by FirstRanker.com ---


Sub-Topic:

A partogram of a woman who is in labor is shown below. Oxytocin infusion is already given.
How wil you manage the condition?


--- Content provided by FirstRanker.com ---


O1:

Forceps-assisted delivery

O2:

--- Content provided by FirstRanker.com ---

Vacuum-a ssisted delivery

O3:

Oxytocin infusion

--- Content provided by FirstRanker.com ---

O4:

Cesarean section

Ans: 4

Ques No: 143

--- Content provided by FirstRanker.com ---


Subject:
Pharmacology

Topic:
CVS Pharmacology

--- Content provided by FirstRanker.com ---

Sub-Topic:

The mechanism of action of tissue plasminogen activator is

O1:

Inhibit extrinsic pathway

--- Content provided by FirstRanker.com ---



O2:

Inhibits platelet aggregation

O3:

--- Content provided by FirstRanker.com ---

Enhance fibrin degradation

O4:

Inhibit clot formation

--- Content provided by FirstRanker.com ---

Ans: 3

Ques No: 144

Subject:
Gynaecology & Obstetrics

--- Content provided by FirstRanker.com ---

Topic:
Obstetrics

Sub-Topic:

A woman presents to you at 36 weeks of gestation with complaints of breathlessness and
excessive abdominal distension. Fetal movements are normal. On examination, fetal parts are

--- Content provided by FirstRanker.com ---

not easily felt and fetal heartbeat is heard but it is muf led. Her symphysis fundal height is 41
cm. Her abdomen is tense but not tender. What is the most likely diagnosis?

O1:

Abruptio placenta

--- Content provided by FirstRanker.com ---


O2:

Hydrocephalus of fetus

O3:

--- Content provided by FirstRanker.com ---

Polyhydramnios

O4:

Fetal-maternal ascites

--- Content provided by FirstRanker.com ---

Ans: 3

Ques No: 145

Subject:
Gynaecology & Obstetrics

--- Content provided by FirstRanker.com ---

Topic:
Gyne cology

Sub-Topic:

For which of the fol owing procedures in the given OT list, can you preferably do a
hysteroscopy?

--- Content provided by FirstRanker.com ---


O1:

Tubal ligation

O2:

--- Content provided by FirstRanker.com ---

Asherman syndrome

O3:

Endocervical polyp

--- Content provided by FirstRanker.com ---

O4:

Subserosal fibroid


Ans: 2

Ques No: 146

--- Content provided by FirstRanker.com ---


Subject:
Gynaecology & Obstetrics

Topic:
Obstetrics

--- Content provided by FirstRanker.com ---

Sub-Topic:

A primigravida presents to you with anemia early in her pregnancy. She is 7 weeks pregnant
as seen on ultrasound. Her hemoglobin level is 9 g/dL. When should the iron supplements be
started for her?

--- Content provided by FirstRanker.com ---

O1:

10 to 12 weeks

O2:

8 to 10 weeks

--- Content provided by FirstRanker.com ---


O3:

After 14 weeks

O4:

--- Content provided by FirstRanker.com ---

After 20 weeks

Ans: 2

Ques No: 147

--- Content provided by FirstRanker.com ---

Subject:
Gynaecology & Obstetrics

Topic:
Gynecology

Sub-Topic:

--- Content provided by FirstRanker.com ---

A 23-year-old woman accompanied by her mother-in-law comes to the infertility clinic. She has
been having regular intercourse for 6 months but is not able to conceive. What is the next best
step?

O1:

--- Content provided by FirstRanker.com ---

Semen analysis for husband

O2:

Reassure a nd review the couple after 6 months

--- Content provided by FirstRanker.com ---

O3:

Hysterolaparoscopy

O4:

Diagnostic hysteroscopy

--- Content provided by FirstRanker.com ---


Ans: 2

Ques No: 148

Subject:
Anaesthesia

--- Content provided by FirstRanker.com ---


Topic:
Cardiopulmonary Cerebral Resuscitation

Sub-Topic:

During resuscitation, when is the given position indicated?

--- Content provided by FirstRanker.com ---




O1:

Unconsciousness with pulse and breathing absent

--- Content provided by FirstRanker.com ---

O2:

Unconsciousness with pulse present and breathing absent

O3:

Unconsciousness with pulse and breathing present

--- Content provided by FirstRanker.com ---


O4:

Unconsciousness with pulse absent and breathing present

Ans: 3

--- Content provided by FirstRanker.com ---

Ques No: 1 49

Subject:
Gynaecology & Obstetrics

Topic:
Gynecology

--- Content provided by FirstRanker.com ---


Sub-Topic:

A 50-year-old woman presents with foul-smel ing bloody discharge per vagina mixed with
mucous. On examination, a necrotizing growth is seen in the cervix with lateral parametrium
involvement. What is the management for this patient?

--- Content provided by FirstRanker.com ---


O1:

Chemotherapy

O2:

--- Content provided by FirstRanker.com ---

Brachytherapy

O3:

Chemoradiation


--- Content provided by FirstRanker.com ---

O4:

Surgery

Ans: 3

Ques No: 150

--- Content provided by FirstRanker.com ---


Subject:
Gynaecology & Obstetrics

Topic:
Gynecology

--- Content provided by FirstRanker.com ---

Sub-Topic:

A 28-year-old woman with a history of multiple sexual partners presents with lower abdominal
pain for 1 month. She experiences minimal discharge and complaints of intermenstrual
bleeding. What is the most likely cause?

--- Content provided by FirstRanker.com ---

O1:

Herpes simplex

O2:

Genital tuberculosis

--- Content provided by FirstRanker.com ---


O3:

Candida

O4:

--- Content provided by FirstRanker.com ---

Neisseria gonorrhoea

Ans: 2

Ques No: 151

--- Content provided by FirstRanker.com ---

Subject:
Gynaecology & Obstetrics

Topic:
Gynecology

Sub-Topic:

--- Content provided by FirstRanker.com ---

A woman is set to undergo a complete laparoscopic hysterectomy. She wants to know if there
are any disadvantages to this procedure. What are the disadvantages of laproscopis
over-open surgery?

O1:

--- Content provided by FirstRanker.com ---

Prolonged recovery time

O2:

Not knowing the extent of thermal burns

--- Content provided by FirstRanker.com ---

O3:

Increased bleeding

O4:

Increased pain

--- Content provided by FirstRanker.com ---


Ans: 2

Ques No: 152

Subject:
Gynaecology & Obstetrics

--- Content provided by FirstRanker.com ---


Topic:
Obstetrics

Sub-Topic:



--- Content provided by FirstRanker.com ---

A 27-year-old pregnant G3P2L2 woman presents to you at 36+6 weeks. Ultrasound is done
and shows the fetus is in a transverse lie. The liquor is adequate, the placenta is normal, and
she has no risk factors. Both her previous deliveries were normal vaginal deliveries. How wil
you manage this patient?

--- Content provided by FirstRanker.com ---

O1:

Cesarean section

O2:

External cephalic version

--- Content provided by FirstRanker.com ---


O3:

Expectant management

O4:

--- Content provided by FirstRanker.com ---

Induction of labor

Ans: 2

Ques No: 153

--- Content provided by FirstRanker.com ---

Subject:
Gynaecology & Obstetrics

Topic:
Gynecology

Sub-Topic:

--- Content provided by FirstRanker.com ---

A female presents to you with 6 weeks of amenorrhea complaining of bleeding per vagina and
slight abdominal pain. The urine pregnancy test is positive and hCG level is 2800 IU/L. A mass
is seen on the left adnexa measuring 3 x 2.5 cm. She is hemodynamical y stable. How wil you
manage this patient?
hCG-Human chorionic gonadotropin

--- Content provided by FirstRanker.com ---


O1:

Oral methotrexate

O2:

--- Content provided by FirstRanker.com ---

Single-dose methotrexate injection

O3:

Serial meth otrexate + leucovorin rescue

--- Content provided by FirstRanker.com ---

O4:

Salpingectomy

Ans: 2

Ques No: 154

--- Content provided by FirstRanker.com ---


Subject:
Gynaecology & Obstetrics

Topic:
Gynecology

--- Content provided by FirstRanker.com ---

Sub-Topic:

A 24-year-old woman who is being treated for infertility with human menopausal gonadotropin
came with complaints of sudden abdominal pain, nausea, vomiting, and breathlessness. The
finding of the ultrasound is shown below. What is the most likely diagnosis?


--- Content provided by FirstRanker.com ---


O1:

Ovarian hyperstimulation syndrome

O2:

--- Content provided by FirstRanker.com ---

Polycystic ovarian syndrome

O3:

Theca lutein cysts

--- Content provided by FirstRanker.com ---

O4:

Granulosa cel tumor

Ans: 1

Ques No: 155

--- Content provided by FirstRanker.com ---


Subject:
Gynaecology & Obstetrics

Topic:
Gyne cology

--- Content provided by FirstRanker.com ---

Sub-Topic:

During a hysteroscopic removal of a submucosal fibroid, a fluid deficit of 2000 mL is estimated
in a patient. What is the immediate complication of this?

O1:

--- Content provided by FirstRanker.com ---

Acute tubular necrosis

O2:

DIC

--- Content provided by FirstRanker.com ---

O3:

Pulmonary edema

O4:

Thromboembolism

--- Content provided by FirstRanker.com ---


Ans: 3



Ques No: 156

--- Content provided by FirstRanker.com ---

Subject:
Gynaecology & Obstetrics

Topic:
Gynecology

Sub-Topic:

--- Content provided by FirstRanker.com ---

A woman comes with complaints of pain and swel ing in the perineal area. She also has
complaints of dif iculty in walking and sitting. She gives a history of multiple sexual partners.
On examination, a tender swel ing is seen with redness on the labia, as shown in the image
given below. What is the most likely diagnosis?

--- Content provided by FirstRanker.com ---

O1:

Chlamydial infection

O2:

Bartholin abscess

--- Content provided by FirstRanker.com ---


O3:

Genital Tub erculosis

O4:

--- Content provided by FirstRanker.com ---

Herpes infection

Ans: 2

Ques No: 157

--- Content provided by FirstRanker.com ---

Subject:
Gynaecology & Obstetrics

Topic:
Gynecology

Sub-Topic:

--- Content provided by FirstRanker.com ---

A 54-year-old woman was diagnosed with advanced cervical cancer. She has a 14-year-old
daughter. What advice would you give her daughter?

O1:



--- Content provided by FirstRanker.com ---

Advise HPV vaccine

O2:

Screen for BRCA mutation

--- Content provided by FirstRanker.com ---

O3:

Screen for PTEN mutation

O4:

Perform cervical biopsy

--- Content provided by FirstRanker.com ---


Ans: 1

Ques No: 158

Subject:
Gynaecology & Obstetrics

--- Content provided by FirstRanker.com ---


Topic:
Gynecology

Sub-Topic:

A 17-year-old girl is seen for primary amenorrhea. There is no development of breasts or hair

--- Content provided by FirstRanker.com ---

in the pubic or axil ary region. Her height is 155 cm, and her weight is 48 kg. She has bilateral
inguinal masses. The uterus, fal opian tube, and Ovary are absent on ultrasound examination.
What is the most likely diagnosis?

O1:

--- Content provided by FirstRanker.com ---

Complete androgen insensitivity syndrome

O2:

Hypergonadotropic hypogonadism

--- Content provided by FirstRanker.com ---

O3:

Turner syndrome

O4:

Polycystic ovary syndrome

--- Content provided by FirstRanker.com ---


Ans: 2

Ques No: 159

Subject:
Surgery

--- Content provided by FirstRanker.com ---


Topic:
Onco logy

Sub-Topic:

A 54-year-old woman presents with cervical cancer stage 2A. It is decided to give neoadjuvant

--- Content provided by FirstRanker.com ---

chemotherapy. What does neoadjuvant chemotherapy mean?

O1:

Chemotherapy is given along with radiation.

--- Content provided by FirstRanker.com ---

O2:

Chemotherapy is given during surgery.

O3:

Chemotherapy is given before radical surgery to reduce the bulk of the tumor

--- Content provided by FirstRanker.com ---


O4:

Chemotherapy is given after radical surgery for micrometastases


Ans: 3

--- Content provided by FirstRanker.com ---

Ques No: 160

Subject:
Gynaecology & Obstetrics

Topic:
Gynecology

--- Content provided by FirstRanker.com ---


Sub-Topic:

A 16-year-old girl presents with cyclical pelvic pain every month. She has not achieved
menarche yet. On examination, a suprapubic bulge can be seen in the lower abdomen. PR
examination reveals a bulging swel ing in the anterior aspect. What is the most likely

--- Content provided by FirstRanker.com ---

diagnosis?

O1:

Transverse vaginal septum above the vagina

--- Content provided by FirstRanker.com ---

O2:

Vaginal atresia

O3:

Imperforate hymen

--- Content provided by FirstRanker.com ---


O4:

Cervical agenesis

Ans: 3

--- Content provided by FirstRanker.com ---

Ques No: 161

Subject:
Gynaecology & Obstetrics

Topic:
Obstetrics

--- Content provided by FirstRanker.com ---


Sub-Topic:

A pregnant woman comes to the clinic. She already has twins by normal delivery. Which of the
fol owing is the correct representation of her obstetric score?

--- Content provided by FirstRanker.com ---

O1:

G3P2

O2:

G3P1

--- Content provided by FirstRanker.com ---


O3:

G2P1

O4:

--- Content provided by FirstRanker.com ---

G2P2

Ans: 3

Ques No: 1
62

--- Content provided by FirstRanker.com ---


Subject:
Surgery

Topic:
Gastrointestinal Surgery

--- Content provided by FirstRanker.com ---

Sub-Topic:

A patient presents with sudden onset of hematemesis. He has been taking aspirin for his
arthritis and drinks alcohol occasional y. He gives a history of occasional abdominal pain. On


examination, there is no abdominal mass or tenderness. What is the likely diagnosis?

--- Content provided by FirstRanker.com ---


O1:

Mal ory-Weiss tear

O2:

--- Content provided by FirstRanker.com ---

Esophagitis

O3:

Peptic ulcer

--- Content provided by FirstRanker.com ---

O4:

Esophageal varices

Ans: 3

Ques No: 163

--- Content provided by FirstRanker.com ---


Subject:
Gynaecology & Obstetrics

Topic:
Obstetrics

--- Content provided by FirstRanker.com ---

Sub-Topic:

A woman presents to you at 36 weeks of gestation with complaints of feeling lightheadedness
and dizziness when she lies on her back. She says she feels alright if she lies on her side or
when walks. What is the most likely reason behind this?
IVC- inferior vena cava

--- Content provided by FirstRanker.com ---


O1:

Increased intracranial pressure

O2:

--- Content provided by FirstRanker.com ---

IVC compression

O3:

Heavy meals

--- Content provided by FirstRanker.com ---

O4:

Excessive venous pooling at the feet

Ans: 2

Ques No: 1 64

--- Content provided by FirstRanker.com ---


Subject:
Surgery

Topic:
Oncology

--- Content provided by FirstRanker.com ---

Sub-Topic:

A 40-year-old male patient presents with a swel ing that has been slowly growing in the past 2
years. On examination, it was variable in consistency and ful y mobile. What is the most likely
diagnosis?

--- Content provided by FirstRanker.com ---

O1:

Dermoid cyst



O2:

--- Content provided by FirstRanker.com ---

Parotid tumor

O3:

Sebaceous cyst

--- Content provided by FirstRanker.com ---

O4:

Cervical lymph node

Ans: 2

Ques No: 165

--- Content provided by FirstRanker.com ---


Subject:
Radiology

Topic:
Radiotherapy

--- Content provided by FirstRanker.com ---

Sub-Topic:

A woman with endometrial carcinoma is undergoing radiotherapy. Which of the fol owing is
true?

O1:

--- Content provided by FirstRanker.com ---

Intensity is i nversely proportional to the square of the distance from the source

O2:

Smal blood vessels are most radioresistant

--- Content provided by FirstRanker.com ---

O3:

Rapidly proliferating cel s are most radioresistant

O4:

Smal intestinal mucosa is most radio-resistant

--- Content provided by FirstRanker.com ---


Ans: 1

Ques No: 166

Subject:
Forensic Medicine

--- Content provided by FirstRanker.com ---


Topic:
Forensic Toxicology

Sub-Topic:



--- Content provided by FirstRanker.com ---

A child before playing consumed fruit from the garden. After some time he developed a high
fever, confusion, photophobia, and unable to urinate. What are the likely causative agent and
the appropriate antidote used in this case?

O1:

--- Content provided by FirstRanker.com ---

Datura, Pralidoxime

O2:

Datura, Physostigmine

--- Content provided by FirstRanker.com ---

O3:

Yel ow oleander, Pralidoxime

O4:

Yel ow oleander, Physostigmine

--- Content provided by FirstRanker.com ---


Ans: 2

Ques No: 167

Subject:
Radiology

--- Content provided by FirstRanker.com ---


Topic:
Thoracic Radiology

Sub-Topic:

A patient presents with a history of fever and cough for the past 3-5 days. His examination

--- Content provided by FirstRanker.com ---

findings reveal crepitations. Chest X-ray is given below. What is the probable diagnosis?

O1:

Apical segment of Right lower lobe consolidation

--- Content provided by FirstRanker.com ---

O2:

Right middle lobe consolidation

O3:

Posterior segment of right lower lobe consolidation

--- Content provided by FirstRanker.com ---




O4:

Loculated pleural ef usion

--- Content provided by FirstRanker.com ---

Ans: 2

Ques No: 168

Subject:
Anatomy

--- Content provided by FirstRanker.com ---

Topic:
Lower Limb

Sub-Topic:

Injury at which of the fol owing marked sites causes failure of dorsiflexion?

--- Content provided by FirstRanker.com ---

O1:

3

O2:

2

--- Content provided by FirstRanker.com ---


O3:

1

O4:

--- Content provided by FirstRanker.com ---

4

Ans: 1

Ques No: 1
69

--- Content provided by FirstRanker.com ---


Subject:
Anatomy

Topic:
Head and Neck

--- Content provided by FirstRanker.com ---

Sub-Topic:

A patient presents with swel ing under the left ear lobule and complains of pain. Which of the
fol owing structures is the likely cause of the pain?



--- Content provided by FirstRanker.com ---

O1:

Facial nerve

O2:

Greater auricular nerve

--- Content provided by FirstRanker.com ---


O3:

Auriculotemporal nerve

O4:

--- Content provided by FirstRanker.com ---

Investing layer of deep cervical fascia

Ans: 2

Ques No: 170

--- Content provided by FirstRanker.com ---

Subject:
Biochemistry

Topic:
Lipid Metabolism

Sub-Topic:

--- Content provided by FirstRanker.com ---

A child presents with bone pain and hepatosplenomegaly. A trephine biopsy and aspirate
show the fol owing finding. Which of the fol owing is the most likely enzyme deficient in this
condition?

O1:

--- Content provided by FirstRanker.com ---

Hexosamini dase

O2:

Glucocerebrosidase

--- Content provided by FirstRanker.com ---

O3:

Sphingomyelinase

O4:

Alpha 1,4-glucosidase

--- Content provided by FirstRanker.com ---


Ans: 2

Ques No: 171

Subject:
Forensic Medicine

--- Content provided by FirstRanker.com ---


Topic:
Forensic Thanatology


Sub-Topic:

The method of autopsy carried out en masse to remove from tongue to prostate is.

--- Content provided by FirstRanker.com ---


O1:

Virchow technique

O2:

--- Content provided by FirstRanker.com ---

Rokitansky technique

O3:

Ghon technique

--- Content provided by FirstRanker.com ---

O4:

Letul e technique

Ans: 4

Ques No: 172

--- Content provided by FirstRanker.com ---


Subject:
Forensic Medicine

Topic:
Forensic Thanatology

--- Content provided by FirstRanker.com ---

Sub-Topic:

A 42-year-old HIV-positive patient died, and his body was brought for post-mortem. The
method of autopsy to be done is___

O1:

--- Content provided by FirstRanker.com ---

Ghon technique

O2:

Letul e technique

--- Content provided by FirstRanker.com ---

O3:

Virchow technique

O4:

Rokitansky technique

--- Content provided by FirstRanker.com ---


Ans: 4

Ques No: 1 73

Subject:
Microbiology

--- Content provided by FirstRanker.com ---


Topic:
Virology

Sub-Topic:

A patient presented with a high fever, altered sensorium, headache, neck rigidity, and

--- Content provided by FirstRanker.com ---

seizures. He was diagnosed with HSV encephalitis. Which of the fol owing tests can be used
to confirm the diagnosis?

O1:

IgM in CSF

--- Content provided by FirstRanker.com ---


O2:

Tzanck smear of CSF

O3:

--- Content provided by FirstRanker.com ---

CSF PCR for viral DNA


O4:

CSF culture on chick embryo lines

--- Content provided by FirstRanker.com ---

Ans: 3

Ques No: 174

Subject:
Microbiology

--- Content provided by FirstRanker.com ---

Topic:
Parasitology

Sub-Topic:

In a patient presented with a fever and a positive filarial antigen test, what is the next
appropriate method of management?

--- Content provided by FirstRanker.com ---


O1:

Bone marrow biopsy

O2:

--- Content provided by FirstRanker.com ---

DEC provocation test

O3:

Detection of microfilariae in the blood smear

--- Content provided by FirstRanker.com ---

O4:

Ultrasound of the scrotum

Ans: 3

Ques No: 175

--- Content provided by FirstRanker.com ---


Subject:
Forensic Medicine

Topic:
Sexual Jurisprudence

--- Content provided by FirstRanker.com ---

Sub-Topic:

A 10-year-old male child came to the casualty with dif iculty in walking and pain in the perianal
region. On subjecting the specimen from the perianal region to a test, it produces yel ow
needle-shaped rhombic crystals with picric acid. What is the test done?

--- Content provided by FirstRanker.com ---

O1:

Barberio te st

O2:

Florence test

--- Content provided by FirstRanker.com ---


O3:

Teichmann test

O4:

--- Content provided by FirstRanker.com ---

Acid phosphatase test

Ans: 1

Ques No: 176

--- Content provided by FirstRanker.com ---

Subject:
Ophthalmology

Topic:
Optics

Sub-Topic:

--- Content provided by FirstRanker.com ---



Which of the fol owing refractive errors is associated with the image given below?

O1:

Presbyopia

--- Content provided by FirstRanker.com ---


O2:

Astigmatism

O3:

--- Content provided by FirstRanker.com ---

Hypermetropia

O4:

Myopia

--- Content provided by FirstRanker.com ---

Ans: 2

Ques No: 177

Subject:
Pathology

--- Content provided by FirstRanker.com ---

Topic:
Gen etics

Sub-Topic:

Which of the fol owing is true regarding the Mendelian mode of inheritance?

--- Content provided by FirstRanker.com ---

O1:

Every 10th patient has a Y-linked disorder

O2:

1/3rd of Mendelian disorders is autosomal recessive

--- Content provided by FirstRanker.com ---


O3:

Every 5th patient has an X-linked recessive disorder

O4:

--- Content provided by FirstRanker.com ---

2/3 rd of Mendelian disorders are autosomal dominant

Ans: 4

Ques No: 178


--- Content provided by FirstRanker.com ---


Subject:
Forensic Medicine

Topic:
Forensic Toxicology

--- Content provided by FirstRanker.com ---

Sub-Topic:

A patient was brought to casualty with seizures. He had multiple episodes and was taking
herbal medicines for the same. He was conscious in between the episodes. During the
episodes, he had arching of his back. What is the likely cause?

--- Content provided by FirstRanker.com ---

O1:

Strychnine

O2:

Ricinus

--- Content provided by FirstRanker.com ---


O3:

Nerium odorum

O4:

--- Content provided by FirstRanker.com ---

Datura

Ans: 1

Ques No: 179

--- Content provided by FirstRanker.com ---

Subject:
Ophthalmology

Topic:
Glaucoma

Sub-Topic:

--- Content provided by FirstRanker.com ---

An infant is brought with complaints of excessive watering of the eyes and photophobia. The
image is given below. What is the likely diagnosis?

O1:

Congenital

--- Content provided by FirstRanker.com ---

glaucoma

O2:

Congenital cataract

--- Content provided by FirstRanker.com ---

O3:

Mucopolysaccharidosis

O4:



--- Content provided by FirstRanker.com ---

Ophthalmia neonatorum

Ans: 1

Ques No: 180

--- Content provided by FirstRanker.com ---

Subject:
Ophthalmology

Topic:
Optics

Sub-Topic:

--- Content provided by FirstRanker.com ---

A 60-year-old patient complains of decreased distant vision but now he does not use
spectacles for near vision. The image of his ocular examination is given below. What is this
type of refractive error cal ed?

O1:

--- Content provided by FirstRanker.com ---

Index myopia

O2:

Axial myopia

--- Content provided by FirstRanker.com ---

O3:

Curvatural myopia




--- Content provided by FirstRanker.com ---

O4:

Positional myopia

Ans: 1

Ques No: 181

--- Content provided by FirstRanker.com ---


Subject:
Pediatrics

Topic:
Pediatric Respiratory Disorders

--- Content provided by FirstRanker.com ---

Sub-Topic:

A child is brought to the hospital with respiratory distress and biphasic stridor. The radiograph
is shown below. What is the diagnosis?

O1:

--- Content provided by FirstRanker.com ---

Acute epiglo
ttitis

O2:

Acute laryngotracheobronchitis

--- Content provided by FirstRanker.com ---


O3:

Foreign body aspiration

O4:

--- Content provided by FirstRanker.com ---



Laryngomalacia

Ans: 2

Ques No: 182

--- Content provided by FirstRanker.com ---


Subject:
Pathology

Topic:
Hematology: Red Blood Cel s

--- Content provided by FirstRanker.com ---

Sub-Topic:

A child presents with intermittent jaundice and splenomegaly. There is a history of similar
complaints in the elder brother. Peripheral smear shows the following finding. How wil you
investigate this condition?

--- Content provided by FirstRanker.com ---

O1:

Osmotic fragility test - Hereditary spherocytosis

O2:

Coombs te st - AIHA

--- Content provided by FirstRanker.com ---


O3:

G6PD deficiency - Genetic testing

O4:

--- Content provided by FirstRanker.com ---

Paroxysmal nocturnal hemoglobinuria - Flow cytometry

Ans: 1

Ques No: 183

--- Content provided by FirstRanker.com ---

Subject:
Pediatrics

Topic:
Pediatric Gastroenterology

Sub-Topic:

--- Content provided by FirstRanker.com ---

A 3-month-old baby presents with jaundice and clay-coloured stools. Lab investigation reveals
that the baby has conjugated hyperbilirubinemia. The liver biopsy shows periductal


proliferation. What is the most likely diagnosis?

--- Content provided by FirstRanker.com ---

O1:

Crigler-Naj ar syndrome

O2:

Rotor syndrome

--- Content provided by FirstRanker.com ---


O3:

Dubin-Johnson syndrome

O4:

--- Content provided by FirstRanker.com ---

Biliary atresia

Ans: 4

Ques No: 184

--- Content provided by FirstRanker.com ---

Subject:
Forensic Medicine

Topic:
Sexual Jurisprudence

Sub-Topic:

--- Content provided by FirstRanker.com ---

Section 314 IPC deals with

O1:

Causing miscarriage with the consent of the mother

--- Content provided by FirstRanker.com ---

O2:

Causing miscarriage without the consent of the mother

O3:

Death of the mother by act done with intent to cause miscarriage

--- Content provided by FirstRanker.com ---


O4:

Causing the death of the quick unborn child by an act amounting to culpable homicide

Ans: 3

--- Content provided by FirstRanker.com ---

Ques No: 185

Subject:
Pathology

Topic:
He matology: White Blood Cels

--- Content provided by FirstRanker.com ---


Sub-Topic:

An African child is found to have a mass in the jaw. A biopsy of the mass was done, and it
appears as shown in the image below. The child also had a translocation of t(8;14). What is
the probable diagnosis?

--- Content provided by FirstRanker.com ---




O1:

Acute myeloid leukemia

--- Content provided by FirstRanker.com ---

O2:

Fol icular lymphoma

O3:

Multiple myeloma

--- Content provided by FirstRanker.com ---


O4:

Burkitt lymphoma

Ans: 4

--- Content provided by FirstRanker.com ---

Ques No: 186

Subject:
Pediatrics

Topic:
Infectious Diseases

--- Content provided by FirstRanker.com ---


Sub-Topic:

An infant presents with hepatosplenomegaly and thrombocytopenia. Neuroimaging with CT
shows peri ventricular calcifications. What is the most likely diagnosis?

--- Content provided by FirstRanker.com ---

O1:

Congenital rubel a syndrome

O2:

Congenital herpes simplex virus infection

--- Content provided by FirstRanker.com ---


O3:

Congenital toxoplasmosis

O4:

--- Content provided by FirstRanker.com ---

Congenital cytomegalovirus infection

Ans: 4

Ques No: 187

--- Content provided by FirstRanker.com ---

Subject:
Forensic Medicine


Topic:
Forensic Thanatology

Sub-Topic:

--- Content provided by FirstRanker.com ---

The temperature of a body of a deceased person is found to be 39 degrees Celsius. Which of
the fol owing is the most probable reason?

O1:

Cyanide poisoning

--- Content provided by FirstRanker.com ---


O2:

Septicemia

O3:

--- Content provided by FirstRanker.com ---

Corrosive poisoning

O4:

Intra-abdominal hemorrhage

--- Content provided by FirstRanker.com ---

Ans: 2

Ques No: 188

Subject:
Pathology

--- Content provided by FirstRanker.com ---

Topic:
Renal System

Sub-Topic:

A child presented to the hospital with cola-colored urine, hypertension, and puf iness of eyes.
Laboratory investigations were done, and creatinine was 2.5 mg/dL.

--- Content provided by FirstRanker.com ---

Treatment was started, and despite treatment, the patient did not improve for the next 3
weeks. The creatinine value increased to 4.5 mg/dL. Which among the fol owing electron
microscopic findings wil be seen in this patient?

O1:

--- Content provided by FirstRanker.com ---

Subendothelial deposits

O2:

Subepithelial deposits

--- Content provided by FirstRanker.com ---

O3:

Crescent fo rmation

O4:

Mesangial deposits

--- Content provided by FirstRanker.com ---


Ans: 2

Ques No: 189

Subject:
PSM

--- Content provided by FirstRanker.com ---


Topic:
Communicable and Non-communicable Diseases

Sub-Topic:

A 35-year-old homeless man presented with a 1-month history of fever, cough, and weight

--- Content provided by FirstRanker.com ---

loss. Both sputum smears turned out to be negative, but the chest x-ray ordered was
suggestive of tuberculosis. According to the recent NTEP guidelines, which is the next best
line of management?


O1:

--- Content provided by FirstRanker.com ---

Repeat sputum smears

O2:

Ask for CBNAAT

--- Content provided by FirstRanker.com ---

O3:

Ask for line probe assay

O4:

Wait until TB culture results to start ATT

--- Content provided by FirstRanker.com ---


Ans: 2

Ques No: 190

Subject:
Forensic Medicine

--- Content provided by FirstRanker.com ---


Topic:
Forensic Toxicology

Sub-Topic:

A farmer was sleeping in the field, and he felt a sting on his leg. He saw something moving

--- Content provided by FirstRanker.com ---

away quickly. He then got drowsy and was taken to the hospital. He developed pain around
the site and continued to bleed profusely from the wound site. The wound became red with
blisters. Which of the fol owing is the most likely cause?

O1:

--- Content provided by FirstRanker.com ---

Viper

O2:

Cobra

--- Content provided by FirstRanker.com ---

O3:

Wasp bite

O4:

Scorpion

--- Content provided by FirstRanker.com ---


Ans: 1

Ques No: 191

Subject:
Pa thology

--- Content provided by FirstRanker.com ---


Topic:
Blood Banking and Transfusion Medicine

Sub-Topic:

A patient was transfused 4 units of blood 2 hours ago, and now she complains of chest

--- Content provided by FirstRanker.com ---

discomfort. X-ray shows bilateral fissural thickening. Which of the fol owing is the most
probable cause?

O1:

ABO incompatibility reactions

--- Content provided by FirstRanker.com ---


O2:

Fluid overload

O3:

--- Content provided by FirstRanker.com ---

Hypocalcemia

O4:



Graft-versus-host disease

--- Content provided by FirstRanker.com ---


Ans: 2

Ques No: 192

Subject:
PSM

--- Content provided by FirstRanker.com ---


Topic:
Epidemiology

Sub-Topic:

Which of the fol owing is the sensitive indicator to assess the availability, utilization, and

--- Content provided by FirstRanker.com ---

ef ectiveness of healthcare in a community?

O1:

Infant mortality rate

--- Content provided by FirstRanker.com ---

O2:

Maternal mortality rate

O3:

Immunization coverage

--- Content provided by FirstRanker.com ---


O4:

Disability-adjusted life years

Ans: 1

--- Content provided by FirstRanker.com ---

Ques No: 193

Subject:
Pathology

Topic:
Renal System

--- Content provided by FirstRanker.com ---


Sub-Topic:

A 48-year-old man presents with complaints of facial puf iness, frothy urine, and hypertension.
He gives a history of infection with hepatitis B. Urine examination reveals microscopic
hematuria. The histopathological image of the kidney biopsy shows a spike and dome pattern.

--- Content provided by FirstRanker.com ---

What is the diagnosis of this condition?

O1:

Minimal cha nge disease

--- Content provided by FirstRanker.com ---

O2:

Membranous nephropathy

O3:

FSGS

--- Content provided by FirstRanker.com ---


O4:

PSGN-associated disease

Ans: 2

--- Content provided by FirstRanker.com ---

Ques No: 194

Subject:
Pathology

Topic:
CVS, Blood Vessels and Vasculitis

--- Content provided by FirstRanker.com ---


Sub-Topic:



A 5-year-old child is brought with fever, fatigue, and left ventricular dysfunction. An
endomyocardial biopsy is shown below. What is the probable diagnosis?

--- Content provided by FirstRanker.com ---


O1:

Acute rheumatic fever

O2:

--- Content provided by FirstRanker.com ---

Chagas disease

O3:

Pyogenic myocarditis

--- Content provided by FirstRanker.com ---

O4:

Lymphocytic myocarditis

Ans: 4

Ques No: 195

--- Content provided by FirstRanker.com ---


Subject:
Pathology

Topic:
Respiratory System

--- Content provided by FirstRanker.com ---

Sub-Topic:

An elderly male is known as a smoker presented with chronic cough, significant weight loss,
and fatigue. Serum calcium level is raised. A lung biopsy was done, and it showed large
atypical cel s with hyperchromasia. What is the probable diagnosis?

--- Content provided by FirstRanker.com ---

O1:

Large cel neuroendocrine tumor

O2:

Smal cel carcinoma

--- Content provided by FirstRanker.com ---


O3:

Adenocarcinoma

O4:

--- Content provided by FirstRanker.com ---

Squamous cel carcinoma

Ans: 4

Ques No: 196

--- Content provided by FirstRanker.com ---

Subject:
Pathology



Topic:
Hematology: White Blood Cel s

--- Content provided by FirstRanker.com ---

Sub-Topic:

An elderly male went for a medical examination. Laboratory investigations showed a marked
elevation of lymphocytes with marked leukocytosis. The peripheral smear of the patient is
given below. What is the appropriate next step for this patient?

--- Content provided by FirstRanker.com ---

O1:

Stem cel transplantation

O2:

Bone marrow biopsy

--- Content provided by FirstRanker.com ---


O3:

Flow cytometry

O4:

--- Content provided by FirstRanker.com ---

Start high-dose chemotherapy

Ans: 3

Ques No: 1 97

--- Content provided by FirstRanker.com ---

Subject:
Pathology

Topic:
Platelets and Blood Transfusion

Sub-Topic:

--- Content provided by FirstRanker.com ---

A 25-year-old male presented with low hemoglobin, platelet count of 25,000/mL, gum
bleeding, and raised PT & aPTT. The image of his peripheral smear is given below. What is
the likely diagnosis?

O1:

--- Content provided by FirstRanker.com ---

AML with t(8;21)

O2:

AML with inv(16)


--- Content provided by FirstRanker.com ---


O3:

AML with t(15;17)

O4:

--- Content provided by FirstRanker.com ---

AML with t(11;22)

Ans: 3

Ques No: 198

--- Content provided by FirstRanker.com ---

Subject:
Pathology

Topic:
Platelet and Bleeding Disorders

Sub-Topic:

--- Content provided by FirstRanker.com ---

A young patient presents with a large retroperitoneal hemorrhage. He also gives a frequent
history of intermittent swel ing of knees on strenuous exercise. There is no history of any
mucosal ble eding. Which among the folowing clotting factors is responsible for the condition
mentioned above?

--- Content provided by FirstRanker.com ---

O1:

Factors VIII and IX

O2:

Factors XI and XII

--- Content provided by FirstRanker.com ---


O3:

Von Wil ebrand factor

O4:

--- Content provided by FirstRanker.com ---

Lupus anticoagulant

Ans: 1

Ques No: 199

--- Content provided by FirstRanker.com ---

Subject:
Pathology


Topic:
Respiratory System

Sub-Topic:

--- Content provided by FirstRanker.com ---

A chronic smoker who worked in a cement factory for 20 years developed mesothelioma. The
association is likely to be due to ________.

O1:

Silicosis

--- Content provided by FirstRanker.com ---


O2:

Asbestosis

O3:

--- Content provided by FirstRanker.com ---

Coal workers' pneumoconiosis

O4:

Bagassosis

--- Content provided by FirstRanker.com ---

Ans: 2

Ques No: 200

Subject:
PSM

--- Content provided by FirstRanker.com ---

Topic:
Nutrition and Health

Sub-Topic:

The average daily dietary nutrient intake level suf icient to meet the nutrient requirements of
nearly al (97-98%) healthy individuals in a particular life stage and gender group is known as?

--- Content provided by FirstRanker.com ---


O1:

Adequate intake

O2:

--- Content provided by FirstRanker.com ---

Dietary goal

O3:

Estimated average requirement

--- Content provided by FirstRanker.com ---

O4:

Recommended dietary al owance

Ans: 4


--- Content provided by FirstRanker.com ---

If you wis
h to access the detailed explanations for the correct and

incorrect options of these questions, download the PrepLadder app now.

Download PrepLadder's NEET PG app for Android
Download PrepLadder's NEET PG app for iOS

--- Content provided by FirstRanker.com ---